JEE - MODULE 2 - MATHS - +1 NM Trigonometry & Calculus-1
JEE - MODULE 2 - MATHS - +1 NM Trigonometry & Calculus-1
JEE - MODULE 2 - MATHS - +1 NM Trigonometry & Calculus-1
Published by
Vedantu Innovations Private Limited
D. No. 1081, 3rd Floor, Vistar Arcade,
14th Main Rd, Sector 3, HSR Layout
Bangalore, Karnataka, India 560 102
www.vedantu.com
Vedantu Innovations Private Ltd.
All rights reserved. No part of this book may be reproduced or utilized in any form or
by any means, electronic or mechanical including photocopying, recording or by
any information storage and retrieval system, without permission in writing from the
publishers.
Notice : Vedantu is committed to serving students with the best of the resources and
knowledge. Bearing that in mind, we have obtained all the information in this book
from sources regarded as reliable, and taken utmost care in editing and printing this
book. However, we as authors and publishers are not to be held responsible for unin-
tentional mistakes that might have crept in. Having stated that, errors (if any) brought
to our notice shall be gratefully acknowledged and rectified in upcoming editions.
Printed by
Colours Imprint
475C, Adugodi Main Road,
8th Block, Koramangala, Bengaluru 560 095
www.coloursimprint.com
3
4
Founder’s Message
Dear Student,
It gives me immense pleasure to present to you a Ready Reckoner and Practice booklet by
Vedantu - ‘TATVA’. Tatva in Sanskrit, means a collection of “Core” content/truth and isn’t that
what Vedantu brings to your table - the Core? Vedantu, at all times, ensures that you have
easy accessibility to a collection of essential concepts, theory, derivations, definitions, solved
examples, concept videos, and practice questions, important questions from competitive
examinations and I am pleased to say that all questions in ‘Tatva’ come with detailed solutions.
Tatva is the result of the constant endeavour and research done by our highly experienced
team of teachers and subject experts who compiled relevant content for you so that you come
out with flying colours in IIT JEE/NEET and Olympiads. We strongly believe and vouch for the
effectiveness and relevance of this booklet to grab the desired rank in IIT JEE/ NEET.
Anand Prakash
Founder, Academic Head
Vedantu
Credits
“Happiness lies in the joy of achievement
and the thrill of creative effort.”
—Theodore Roosevelt
Tatva is the brainchild of the creative Vedans who The managers who understood every aspect of what
strived tirelessly to weave success stories for you. Our the leadership were trying to accomplish and brought
heartfelt thanks to the Super Vedans who give wings so much of their own to the table and managed the
to the vision of Vedantu. execution of ‘Tatva’ immaculately.
Our leaders who have been our guiding light and Kiran Kumari Sushmitha M D
encouragement in every step: Harish Rao Akshatha J
Vamsi Sir, Anand Prakash Sir and Pulkit Sir Bhavya Bangera J Charubak Chakrabarti
Shraddha R Shetty
Our gratitude to the insightful leadership and
guidance of our leaders who dreamt about ‘Tatva’,
steered the project in the right direction and were
instrumental in making this dream into a reality:
Sahil Bhatia Sudhanshu Jain
Shubam Gupta Ajay Mittal
Arshad Shahid Jaideep Sontakke
Mathematics Team
Our true appreciation for all the Master Teachers of Assistant Teachers
Vedantu whose relentless efforts helped us in Mohammed Siddiqui Priyadharshini R
accomplishing this vision into reality. Our heartfelt Jyoti Garg Pravin S Rathore
gratitude to our creative content developers, and the Anjulika Gupta Sukamal Chatterjee
DTP team who have put in their hard work, insights, Pankaj Gupta
eagerness to execute and nurture Tatva into ‘your SME
ready handbook’ and to bring positive learning Nikhil Goyal(Team Lead) Aman Bhartiya
experience to you. Vipul Sachdeva
DTP
Raman Kumar
A loud shout out for our media team - a bunch of creative minds with contagious energy. We cannot thank
them enough.
Special thanks and appreciation for the enthusiastic support provided by Arunima Kar, Savin Khandelwal,
Dipshi Shetty and Mohamed Anzar.
The journey of the imagination to the reality of ‘Tatva’ would not have been possible without our enthusiastic
Operations Team, our amazing Academic Team, our dedicated team of Teachers and our talented Tech
Team.
Table of Contents
Theory ............................................................................................................................................... 9
Solved Examples .............................................................................................................................. 23
Exercise - 1 : Basic Objective Questions ......................................................................................... 30
Exercise - 2 : Previous Year JEE Mains Questions .......................................................................... 35
LIMITS
Theory ............................................................................................................................................... 47
Solved Examples ............................................................................................................................... 51
Theory ............................................................................................................................................... 75
Solved Examples ............................................................................................................................... 81
Exercise - 1 : Basic Objective Questions ..........................................................................................91
Exercise - 2 : Previous Year JEE Mains Questions ........................................................................... 97
Exercise - 3 : Advanced Objective Questions ................................................................................. 102
Exercise - 4 : Previous Year JEE Advanced Questions .................................................................... 109
Answer Key ....................................................................................................................................... 115
STATISTICS
MATHEMATICAL REASONING
SETS
A set is a collection of well-defined and well distinguished In this form, we write a variable (say x) representing any
objects of our perception or thought. member of the set followed by a property satisfied by each
member of the set.
1.1 Notations
For example, the set A of all prime numbers less than 10 in
The sets are usually denoted by capital letters A, B, C, etc. the set-builder form is written as
and the members or elements of the set are denoted by lower- A = {x | x is a prime number less that 10}
case letters a, b, c, etc. If x is a member of the set A, we write The symbol '|' stands for the words 'such that'. Sometimes,
x A (read as 'x belongs to A') and if x is not a member of the we use the symbol ':' in place of the symbol '|'.
set A, we write x A (read as 'x does not belong to A,). If x
and y both belong to A, we write x, y A. 3. TYPES OF SETS
2. REPRESENTATION OF A SET
3.1 Empty Set or Null Set
Usually, sets are represented in the following two ways :
A set which has no element is called the null set or empty
(i) Roster form or Tabular form
set. It is denoted by the symbol .
(ii) Set Builder form or Rule Method
2.1 Roster Form For example, each of the following is a null set :
(a) The set of all real numbers whose square is –1.
In this form, we list all the member of the set within braces
(curly brackets) and separate these by commas. For example, (b) The set of all rational numbers whose square is 2.
the set A of all odd natural numbers less that 10 in the Roster (c) The set of all those integers that are both even and odd.
form is written as :
A set consisting of atleast one element is called a
A = {1, 3, 5, 7, 9}
non-empty set.
3.5 Equal Sets The union of two sets A and B, written as A B (read as 'A
union B'), is the set consisting of all the elements which are
Two sets A and B are said to be equals, written as A = B, if
every element of A is in B and every element of B is in A. either in A or in B or in both Thus,
Clearly, x A B x A or x B, and
Two finite sets A and B are said to be equivalent, if n
(A) = n (B). Clearly, equal sets are equivalent but equivalent x A B x A and x B.
sets need not be equal.
For example, the sets A = { 4, 5, 3, 2} and B = {1, 6, 8, 9} are
equivalent but are not equal.
3.7 Subset
P(A) = { , {1}, {2}, {3}, {1,2} {1, 3}, {2, 3}, {1, 2, 3}}
For example, if A = {a, b, c, d) and B = {c, d, e, f}, then
Clearly, if A has n elements, then its power set P (A) contains A B = {c, d}.
exactly 2n elements.
SETS, RELATIONS & FUNCTIONS 11
Two sets A and B are said to be disjoint, if A B = , i.e. A If U is a universal set and A is a subset of U, then the
and B have no element in common. complement of A is the set which contains those elements
of U, which are not contained in A and is denoted by
U A'or Ac. Thus,
Ac = {x : x U and x A}
Important Results
For example, if A = {1, 3, 5} and B = {2, 4, 6},
then A B = , so A and B are disjoint sets.
a) Uc = b) c = U c) A Ac = U
4.4 Difference of Two Sets
d) A Ac =
If A and B are two sets, then their difference A - B is defined
as : 5. ALGEBRA OF SETS
A - B = {x : x A and x B}.
1. For any set A , we have
Similarly, B - A = {x : x B and x A }.
a) A A = A b) A A = A
c) A = A d) A =
e) A U = U f) A U = A
(i) A A B, B A B, A B A, A B B Write the set {x : x is a positive integer and x2 < 30} in the
roster form.
(ii) A - B = A Bc (iii) (A - B) B = A B
(iv) (A - B) B = (v) A B Bc Ac Sol. The squares of positive integers whose squares are less
than 30 are : 1, 2, 3, 4, 5.
(vi) A - B = Bc - Ac (vii) (A B) (A Bc) = A Hence the given set, in roster form, is {1, 2, 3, 4, 5}.
(viii) A B = (A - B) (B - A) (A B)
Example – 4
(ix) A - (A - B) = A B
Write the set {0, 1, 4, 9, 16, .......} in set builder form.
(x) A - B = B -A A= B (xi)A B =A B A= B
(xii) A (B C) = (A B) (A C) Sol. The elements of the given set are squares of integers :
0, 1, 2, 3, 4, .......
Example – 1
Hence the given set, in set builder form, is {x2 : xZ}.
Write the set of all positive integers whose cube is odd.
Example – 5
2k 1 : k 0, k Z .
Sol. (i) A = {1, 2}.
Example – 2 [ x2 – 3x + 2 = 0 (x – 1) (x – 2) = 0 x = 1, 2]
Hence A is finite.
1 2 3 4 5 6 7 (ii) B = {3}.
Write the set , , , , , , in the set
2 3 4 5 6 7 8
[ x2 = 9 x = + 3. But 3 N]
builder form. Hence B is finite.
Example – 6 Example – 8
Which of the following are empty (null) sets ? Are the following pairs of sets equal ? Give reasons.
(i) Set of odd natural numbers divisible by 2 (i) A = {1, 2}, B = {x : x is a solution of x2 + 3x + 2 = 0}
(ii) {x : 3 < x < 4, x N}
(ii) A = {x : x is a letter in the word FOLLOW},
(iii) {x : x2 = 25 and x is an odd integer}
B = {y : y is a letter in the word WOLF}.
(iv) [x : x2 – 2 = 0 and x is rational]
(v) {x : x is common point of any two parallel lines}.
Sol. (i) A = {1, 2}, B = {–2, –1}
Sol. (i) Since there is no odd natural number, which is divisible [ x2 + 3x + 2 = 0 (x + 2) (x + 1) = 0 x = –2, —1]
by 2.
Clearly A B.
it is an empty set.
(ii) A = {F, O, L, L, O, W} = {F, O, L, W}
(ii) Since there is no natural number between 3 and 4.
it is an empty set. B = {W, O, L, F} = {F, O, L, W}.
Find the pairs of equal sets from the following sets, if any, (b) (i) A B (ii) B D (iii) A B C.
giving reasons :
A = {0}, B = {x : x > 15 and x < 5},
C = {x : x – 5 = 0}, D = {x : x2 = 25}, Sol. (a) (i) A B = {1, 2, 3, 4, 5} {3, 4, 5, 6, 7}
Example – 10 Example – 13
If A1 = {2, 3, 4, 5}, A2 = {3, 4, 5, 6}, A3 = {4, 5, 6, 7}, find Let A = {1, 2, 3, 4, 5, 6}, B = {3, 4, 5, 6, 7, 8}. Find
(A – B) (B – A).
Ai and Ai, where i = {1, 2, 3}.
Sol. We have, A = {1, 2, 3, 4, 5, 6} and B = {3, 4, 5, 6, 7, 8}.
Sol. (i) Ai = A1 A 2 A 3 = {2, 3, 4, 5} {3, 4, 5, 6} A – B = {1, 2} and B – A = {7, 8}
{4, 5, 6, 7} (A – B) (B – A) = {1, 2} {7, 8} = {1, 2, 7, 8}.
= {2, 3, 4, 5} {3, 4, 5, 6, 7} = {2, 3, 4, 5, 6, 7}. Some Basis Results about Cardinal Number
(ii) Ai = A1 A 2 A 3 = {2, 3, 4, 5} {3, 4, 5, 6} If A, B and C are finite sets and U be the finite universal set,
then
{4, 5, 6, 7}
(i) n (Ac) = n (U) - n (A)
= {2, 3, 4, 5} {4, 5, 6} = {4, 5}.
(ii) n (A B) = n (A) + n (B) - n (A B)
Example – 11 (iii) n (A B) = n(A) + n (B), where A and B are disjoint non -
empty sets.
Let U = {1, 2, 3, 4, 5, 6, 7, 8, 9}, A = {1, 2, 3, 4},
(iv) n (A Bc) = n (A) - n (A B)
B = {2, 4, 6, 8}. Find :
(v) n (Ac Bc) = n (A B)c = n (U) - n (A B)
C
(i) AC (ii) BC (iii) (AC)C (iv) A B (vi) n (Ac Bc) = n (A B)c = n (U) - n (A B)
(vii) n (A - B) = n (A) - n (A B)
C
Sol. (i) A = Set of those elements of U, which are not in (viii) n (A B) = n (A B) - n (A Bc) - n (Ac B)
A = {5, 6, 7, 8, 9}. (ix) n (A B C) = n (A) + n (B) + n (C) - n (A B) – n(B C) –
(ii) C
B = Set of those elements of U, which are not in n(C A) + n (A B C)
B = {1, 3, 5, 7, 9}. (x) If A1, A2, A3, ... An are disjoint sets , then
(iii) (AC)C = Set of those elements of U, which are not in n (A1 A2 A3 ... An) = n(A1) + n (A2) + n (A3)
A’ = {1, 2, 3, 4} = A. + ... + n(An)
(xi) n (A B) = number of elements which belong to exactly
(iv) A B = {1, 2, 3, 4} {2, 4, 6, 8} = {1, 2, 3, 4, 6, 8}.
one of A or B.
C
A B = Set of those elements of U, which are not in Example – 14
A B = {5, 7, 9}. If A = {1, 2, 3}, B = {4, 5, 6} and C = {7, 8, 9}, verify that
A B C = A B A C .
Example – 12
If U = {x : x is a letter in English alphabet}, Sol. We have, A = {1, 2, 3}, B = {4, 5, 6} and C = {7, 8, 9}.
A = {x : x is a vowel in English alphabet}. A B = {1, 2, 3} {4, 5, 6} = {1, 2, 3, 4, 5, 6} ...(1)
Find AC and (AC)C. A C = {1, 2, 3} {7, 8, 9}
= {1, 2, 3, 7, 8, 9} ...(2)
Sol. (i) Since A = {x : x is a letter in English alphabet}, and B C = {4, 5, 6} {7, 8, 9} = ...(3)
AC is the set of those elements of U, which are not vowels
Now A B C 1, 2, 3 = {1, 2, 3} ...(4)
= {x : x is a consonant in English alphabet}.
(ii) (AC)C is the set of those elements of U, which are not and A B A C = {1, 2, 3, 4, 5, 6} {1, 2, 3, 7, 8, 9}
consonants = {x : x is a vowel = {1, 2, 3} ...(5)
in English alphabet} = A. From (4) and (5), A B C A B A C , which
C C
Hence (A ) = A. verifies the result.
SETS, RELATIONS & FUNCTIONS 15
Example – 15 Example – 17
x Ac
Sol.
Bc Ac ... (1)
x Ac
x Bc [ Bc Ac]
(ii) x B
A B
n(X) = 200
Prove the following :
n(M A I) = n(M) + n(A) + n (I) –
A – B = A – (A B)
n(M A) –n (A I) – n (M I) + n (M A I)
where U is the universal set.
= 35 + 40 + 40 – 20 – 17 – 15 + 5 = 68
Sol. Let x (A – B), where x is arbitrary. (i) Number of students passed in all three examination
Now x (A – B) = 200 – 68 = 132
x A and x B (ii) Number of students failed in IIT or AIEEE
(x A and x A) and x B =n (I A) = n(I) + n(A) – n (I A)
[Note this step] = 40 + 40 – 17 = 63
x A and (x A and x B)
Example – 21
[Associative Law]
In a hostel, 25 students take tea, 20 students take coffee,
x A and x (A B)
15 students take milk, 10 students take both tea and coffee,
x A – (A B)
8 students take both milk and coffee. None of the them
Hence A – B = A – (A B). take tea and milk both and everyone takes atleast one
(ii) Failed in AIEEE or IIT. Let the sets, T and C and set M are the students who drink
tea, coffee and milk respectively. This problem can be solved
by Venn diagram.
= n (T C M)
2.1 Cartesian product of sets A relation ‘f’ from a set A to set B is said to be a function if
every element of set A has one and only one image in set B.
Definition : Given two non-empty sets P & Q. The cartesian
product P × Q is the set of all ordered pairs of elements from Notations
P & Q i.e.
P × Q = {(p, q); p P; q Q} x f y (= f (x))
(Domain) input output (Range)
2.2 Relations (independent variable) (dependent variable)
(i) Expression under even root (i.e. square root, fourth root etc.)
should be non–negative.
Two functions f & g are said to be equal iff
(ii) Denominator 0.
1. Domain of f = Domain of g
(iii) logax is defined when x > 0, a > 0 and a 1.
2. Co-domain of f = Co-domain of g
(iv) If domain of y = f (x) and y = g(x) are D1 and D2 respectively,
then the domain of f (x) ± g(x) or f (x) . g(x) is D1 D2. While 3. f(x) = g(x) x Domain.
Put y = f(x)
1. Question of format :
Q L Q Q quadratic
y ; y ; y L Linear
Q Q L
(a) One-to-One functions are also called Injective
Range is found out by cross-multiplying & creating functions.
a quadratic in ‘x’ & making D 0 (as x R) (b) Onto functions are also called Surjective
2. Questions to find range in which-the given (c) (one-to-one) & (onto) functions are also called
expression y = f(x) can be converted into x (or some Bijective Functions.
function of x) = expression in ‘y’.
Do this & apply method (ii).
SETS, RELATIONS & FUNCTIONS 19
Relations which can not be catagorized as a function 3.4.2 Constant Function : The function f : R R defined
by y = f(x) = c, x R where c is a constant is
called constant function
y
A B
8
Not a function 6
4
As not all elements of set A are associated with some
elements of set B. (violation of– point (i)– definition 2.1) 2
x’ x
–8 –6 –4 –2 2 4 6 8
–2
–4
–6
–8
A B
Not a function one-to-many f (x) = 3
f (x) = | x |
3.4.1 Identity Function : The function f : R R defined
by y = f(x) = x x R is called identity function. Its also known as “Absolute value function’.
Properties of Modulus Function :
y
8 The modulus function has the following properties :
6
4 1. For any real number x, we have x2 x
2
x’ x
–8 –6 –4 –2 2 4 6 8
–2 2. xy x y
–4
–6
–8 3. x y x y
triangle inequality
f (x) = x 4. x y x y
20 SETS, RELATIONS & FUNCTIONS
3.4.4 Signum Function : The function f : R R defined by Properties of Greatest Integer Function :
Y–axis
1
Fractional part of x, denoted by {x} is given by x – [x]. So,
X–axis
O
x 1; 1 x 2
–1
x x x x ; 0 x 1
x 1; 1 x 0
x Y–axis
; x0
Sgn(x) x
0; x 0
(0, 1)
3.4.5 Greatest Integer Function : The function f : R R X–axis
O
defined as the greatest integer less than or equal to x.
It is usually denoted as y = f(x) = [x]
2 Y–axis
1
–2 –1
X–axis (0, 1)
1 2 3
X–axis
–1 O
–2
–3
f (x) = ax, when 0 < a < 1
SETS, RELATIONS & FUNCTIONS 21
f (x) = logax, a > 0, a1 (i) If a > 1, loga x < logay 0 < x <y
Domain : x (0, ) (ii) If 0 < a < 1, loga x < loga y x > y >0
Range : y R
(iii) If a > 1 then logax < p 0 < x < ap
(iv) If a > 1 then logax > p x > ap
Y–axis
(v) If 0 < a < 1 then logax < p x > ap
(vi) If 0 < a < 1 then logax > p 0 < x < ap
X–axis
O
(1, 0)
f (x)= l og a x, when a > 1 If the exponent and the base are on same side of the
unity, then the logarithm is positive.
If the exponent and the base are on different sides of
unity, then the logarithm is negative.
Y–axis
(vi) a l ogcb bl ogca , a, b, c > 0, c 1. Let f : X R be a real valued function and be a scalar. Here
by scalar, we mean a real number. Then the product f is a
function from X to R defined by ( f) (x) = f(x), x X.
Let f and g be two real functions defined from X R where Definition : A function f (x) is said to be periodic function, if there
X R. The quotient of f by g denoted by f /g is a function exists a positive real number T, such that
defined by f x T f x , x R.
Then, f (x) is a periodic function where least positive
f f x value of T is called fundamental period.
x
g g x , provided g(x) 0, x X.
Graphically : If the graph repeats at fixed interval, then function
is said to be periodic and its period is the width of
4.6 Even and Odd Functions that interval.
Some standard results on periodic functions :
Even Function : f (–x) = f (x), x Domain Functions Periods
n n n n
The graph of an even function y = f (x) is symmetric about the (i) sin x, cos x, sec x, cosec x ; if n is even.
y–axis. i.e., (x, y) lies on the graph (–x, y) lies on the graph. 2; (if n is odd or fraction)
(ii) tann x, cotn x ; n is even or odd.
(iii) |sin x|, |cos x|, |tan x|
y = x2 |cot x|, |sec x|, |cosec x|
Y–axis
(iv) x – [x], [.] represents 1
greatest integer function
(–x, y) (x, y) (v) Algebraic functions period does not exist
e.g., x , x2, x3 + 5, ....etc.
X–axis
O Properties of Periodic Function
(i) If f (x) is periodic with period T, then
Graph of an even function
(a) c . f (x) is periodic with period T.
(b) f (x ± c) is periodic with period T.
Odd Function : f (– x) = –f (x), x Domain (c) f (x) ± c is periodic with period T.
where c is any constant.
The graph of an odd function y = f (x) is symmetric about origin
i.e. if point (x, y) is on the graph of an odd function, then (ii) If f (x) is periodic with period T, then
(–x, –y) will also lie on the graph. k f (cx + d) has period T/|c|,
i.e. Period is only affected by coefficient of x
where k, c, d constant.
3
Y–axis y=x (iii) If f1(x), f2(x) are periodic functions with periods T1, T2
(x, y)
respectively, then we have, h(x) = a f1(x) + b f2(x) has
period as, LCM of {T1, T2}
X–axis
O
a c e LCM of a, c, e
(–x, –y) Graph of an odd function (a) LCM of , ,
b d f HCF of b, d, f
(b) LCM of rational and rational always exists.
LCM of irrational and irrational sometime exists.
But LCM of rational and irrational never exists.
e.g., LCM of (2 , 1, 6 ) is not possible as
2 , 6 irrational and 1 rational.
SETS, RELATIONS & FUNCTIONS 23
SOLVED EXAMPLES
x 2x
If 1, y 1 2, 1 find values of x and y.. R x 4, : 4 x 6, x N
3 2 x
x 7
Sol. 1 2 & y 1 1 Sol. R 8, 3 , 9, , 10, 2
3
3
x = 3 and y = 2.
so, domain = {8, 9, 10}
Example – 2
7
range = 3, , 2
If A = {1, 2}, find A × A × A 3
Example – 6
Sol. A × A × A = {(x, y, z), x A, y A, z A}
so, A × A × A = {(1, 1, 1), (1, 1, ), (1, 2, 1), (2, 1, 1), Let A = {1, 2}. List all the relation on A.
Following figure shows a relation between sets P and Q. Since relation R from set A to set A is a subset of A × A
Write this relation in (i) set builder form, (ii) roster form All the relations on A are :
, {(1, 1)}, {(1, 2)}, {(2, 1)}, {(2, 2)}, {(1, 1), (1, 2)},
1
–1
{(1, 1), (2, 1)}, {(1, 1), (2, 2)}, {(1, 2), (2, 1)}, {(1, 2),
1
(2, 2)}, {(2, 1), (2, 2)}, {(1, 1), (1, 2), (2, 1)}, {(1, 1),
–2 4 (1, 2), (2, 2)}, {(1, 1), (2, 1), (2, 2)}, {(1, 2), (2, 1),
–4 16 (2, 2)}, {(1, 1), (1, 2), (2, 1), (2, 2)}.
2
Since n(A × A) = 4, the number of all relations in the set
P 4
A = 2 i.e., 16.
Example – 7
Sol. It is clear, that relation R is “y is the square of x”.
2
(i) In set builder form, R = {(x, y) : y = x , x P, y Q} Find the domain and range of the following functions
x2 1
Let y x 1 as x 1 Sol. Let f(x) = x + 1, g(x) = 2x – 3
x 1
f + g = f(x) + g(x) = (x + 1) + (2x – 3)
x=y–1
= 3x – 2
Clearly, x is not defined when y = 2 as x 1
f – g = f (x) – g(x) = (x + 1) – (2x – 3)
Range = R – {2}.
= x + 1 – 2x + 3 = –x + 4
1
(ii) Let f x x, : x R, x 1 f x
1 x2 f x 1 3
;x
g g x 2x 3 2
1 2
Clearly, f x 2
is not defined when 1 – x = 0
1 x Example – 10
1 x 1y
2
1
x 1
y 1 (i) Find the domain of the function.
y y
(ii) Find the range of the function.
x is defined when y (–, 0) [1, ). (iii) Find the values of f(2), f(3), f(4), f(6).
Range = (–, 0) [1, ).
Sol. (i) Domain : {1, 2, 3, 4, 5}
Example – 8
(ii) Range :
y y
implies x . Since x R, 1 y 0 . x –1 x 2 0
1 y Sol. For f (x) to be defined and x 3, 4
x – 3 x – 4
i.e. y [0, 1). Thus range is [0, 1)
By wavy – curve method the domain of definition of f (x) is
the set
x – , – 2 1, 3 4, .
SETS, RELATIONS & FUNCTIONS 25
Example – 16 Example – 18
Example – 17 1
Sol. f x x2
log10 1 – x
The value of x if log1/2 x log1/3 x is
{as we know; loga x is defined when x, a > 0 and a 1 also
(a) (0, 1] (b) (0, 1) loga 1 = 0}
(c) [0, 1) (d) None of these Thus, log10 (1 – x) exists when, 1 – x > 0 ...(i)
1
Sol. Case I. When x 1 and x > 0. also exists when, 1 – x > 0
log10 1 – x
log1/2 x log1/3 x and 1 – x 1 ...(ii)
x < 1 and x 0 ...(iii)
log x x log x x
also we have x 2 exists when x + 2 0
1 1
log x log x or x –2 ....(iv)
2
3
1
Thus, f x x 2 exists when (iii) and (iv)
log10 1 – x
1 1
both holds true.
log x 2 log x 3
–2 x < 1 and x 0
1 1 x –2, 0 0 , 1
log x 2 log x 3
Example – 20
logx 2 logx 3 where x 1
Find value of ‘x’ so that log|x| |x – 1| 0.
which is only possible, if 0 < x < 1
Case II. When x = 1.
Sol. It is clear that |x| > 0 and |x| 1
log1/2 x = log1/3 x, equality sign holds true.
x 0, –1, 1
Combining the above cases, Also, |x – 1| > 0 x 1,
0 < x 1 or x (0, 1]. Case I : For 0 < |x| < 1
Hence, (a) is correct answer. x –1, 0 0, 1 ...(1)
SETS, RELATIONS & FUNCTIONS 27
and log|x| |x – 1| 0
1
log|x| |x – 1| log|x| 1 so that Case II : For x , |2x – 1| = 2x – 1
2
0 < |x – 1| 1
–1 x – 1 1 and x 1 2x – 1 = 3[x] + 2{x}.
0 x 2 and x 1 2x – 1 = 3(x – {x}) + 2{x}.
x 0 , 1 1, 2 . ...(2) {x} = x + 1
Now 0 {x} < 1
From (1) and (2) we have x (0, 1)
Case II : For |x| > 1 0 x + 1 < 1. –1 x < 0.
x < –1 or x > 1 1
which is not possible since x .
i.e. x – , – 1 1, ...(3) 2
and log|x| |x – 1| 0
1
|x – 1| 1 Hence x = is the only solution.
4
x – 1 1 or x – 1 –1
x 2 or x 0
Example – 23
i.e. x – , 0 2 , ...(4)
Find the domain of definition of the following
From (3) and (4), we find that x – , – 1 2,
function : f x log 1 2x – 3
Hence x 0, 1 – , – 1 2, 2
Example – 21
Sol. For f(x) to be defined log1/2 (2x – 3) 0
If y =3[x] +1 =2[x – 3] +5, then find the value of 2x – 3 1
[x + y], where [.] represents greatest integer function. x2 ....(1)
Also 2x – 3 > 0
Sol. We are given that 3[x] + 1 = 2([x] – 3) + 5
[x] = –2 3
x . ....(2)
y = 3(–2) + 1 = –5 2
Hence [x + y] = [x] + y = – 2 – 5 = – 7 Combining (1) and (2) we get the required values of x.
3
Example – 22 Hence the domain of definition of f(x) is the set , 2
Solve the equation |2x – 1| = 3[x] + 2{x} for x. 2
where [.] represents greatest integer function and {}
represents fraction part function.
Example – 24
Example – 27
x
x1 x2
–1 1 1
Find the range of the function y
2 sin 3x
2
x , x 0 1
(b) f (x) = x |x| 2 We have, y
x , x 0 2 sin 3x
–1 x
x1 1 2y 1
sin 3x
y
–x2 –1
for x to be real
2y1
1 1 (since, –1 sin 3x 1)
By calculus y
Example – 28
1
Yes!! as y > 0 (why??) y
2 sin 3x
Find domain for f (x) = cos sin x .
2y 1 Alternate 2y 1
1 1 1 1
y y
Sol. f (x) = cos sin x is defined, if
–y 2y – 1 y cos (sin x) 0
2y 1 2y 1 As, we know
1 0 1 0
y y –1 sin x 1 for all x
cos 0
3y 1 y 1 (Here, = sin x lies in the 1st and 4th quadrants)
2y – 1 – y & 2y – 1 y 0 0
y y i.e. cos (sin x) 0, for all x
i.e. x R.
1 Thus, domain f (x) R
y y 1
3
Example – 29
Solve for x
1
Range : y , 1 |x| + |x + 4| = 4
3
Sol. |x| + |x + 4| = 4
As we know, |x| + |y| = |x – y|, iff xy 0
x(x + y) 0
O y3 1 Using number line rule,
+ – +
Alternate Method : –4 0
x [–4, 0]
1
y
2 sin 3x Example – 30
we know, –1 sin 3x 1 x x2
Solve x 1 x
1 –sin 3x 1 x 1
1 2 – sin 3x 1
1 1 1 x
Sol. Let f x and g(x) = x
x 1
1 2 sin 3x 3
x x2
f (x) + g(x) x
1 x 1 x 1
Range y , 1
3 Using, | f (x)| + |g(x)| = |f (x) + g(x)|
Inequality changes upon reciprocating as all expressions i.e. f (x) . g(x) 0
across inequality are (positive). x x2
.x 0 0
x 1 x 1
+
–4 1
x {0} (1, )
30 SETS, RELATIONS & FUNCTIONS
1. Which of the following is the empty set? (b) The smallest set of Y is {2, 3, 5, 9}
(c) {x : x is a real number and x2 - 9 = 0} 9. Given the sets A = {1, 2, 3}, B = {3, 4}, C = {4, 5, 6}, then
(a) a null set 13. X and Y are two sets such that n (X) = 17, n(Y) = 23,
n( X Y) = 38 then n ( X Y) is
(b) a singleton set
(a) 4 (b) 2
(c) a finite set
(c) 6 (d) None of these
(d) not a well defined collection
14. If X and Y are two sets such that X Y has 18 elements, X
Operations on sets (Union and Intersection)
has 8 elements and Y has 15 elements; then number of
6. If A = {1, 3, 5 , 7, 9, 11, 1 3, 1 5, 1 7}, elements in X Y are
B = {2, 4, .......,18}and N is the universal set, then
(a) 5 (b) 8
A c ((A B) B c ) is
(c) 6 (d) None of these
(a) A (b) N
15. If S and T are two sets such that S has 21 elements, T has 32
(c) B (d) None of these
elements, and S T has 11 elements, then number of
7. Let A = {x : x is a multiple of 3} and elements S T has
B = {x : x is a multiple of 5}. Then A B is given by
(a) 42 (b) 50
(a) {3, 6, 9...} (b) {5, 10, 15, 20, ...}
(c) 48 (d) None of these
(c) {15, 30, 45, ...} (d) None of these
SETS, RELATIONS & FUNCTIONS 31
16. In a committee 50 people speak French, 20 speak Spanish 25. The value of x for which 12 x – 6 < 0 and 12 – 3 x < 0
and 10 speak both Spanish and French. The number of (a) (b) R
persons speaking at least one of these two languages is (c) R – {0} (d) none of these
(a) 60 (b) 40
x 3 x 1 x 2
(c) 38 (d) None of these 26. The value of x for which x and
4 2 3
17. In a group of 1000 people, there are 750 people who can 2 – x > 2x – 8
speak Hindi and 400 who can speak English .Then number (a) [–1, 10/3] (b) (–1, 10/3)
of persons who can speak Hindi only is (c) R (d) none of these
(a) 300 (b) 400 27. Solve for x :
(c) 600 (d) None of these
4 4
18. In a statistical investigation of 1,003 families of Calcutta, it 2x 1 x 1 x 2 0
(i) 4
was found that 63families had neither a radio nor a T.V, 794 x 2 x 4
families had a radio and 187 had a T.V. The number of families
in that group having both a radio and a T.V is (a) [½, 2) (b) R
(c) (d) (½, 2)
(a) 36 (b) 41
4 3 2
(ii) (x – 2) (x – 3) (x – 4) (1 – x) < 0
(c) 32 (d) None of these
19. If A has 3 elements and B has 6 elements, then the minimum (a) (1, 3) (b) , 1 3,
number of elements in the set A B is
(c) ,1 3, (d) none
(a) 6 (b) 3
1
(c) (d) None of these (iii) x 2
x
20. The set A = {x : x R, x2 = 16 and 2x = 6} equals
(a) 0, (b) R
(a) (b) {14, 3, 4}
(c) (d) 0,
(c) {3} (d) {4}
x2
INEQUALITIES (iv)
x 1
0
23. Find the set of values of 'x' for which the given condition x 7
29. Solution of 2 is
is true (x – 1) (x – 3) (x + 5) > 0 x 3
(a) (–5, 1) (3, ) (b) (–1, 5)
(a) 3, (b) , 13
(c) [–5, 1] [3, ) (d) none of these
(c) (–13, –3) (d) none of these
x 1 2 30. The set of values of x which satisfy the inequations
24. The value for which
x x2
5x + 2 < 3x + 8 and 4 is
x 1
(a) (0, 1) (b) , 1
(a) , 1 (b) (2, 3)
(c) , 0 (d) 1, 0
(c) , 3 (d) , 1 2, 3
32 SETS, RELATIONS & FUNCTIONS
(a) many-one into function 43. The domain of the function f (x) = loge (x – [x]), where [.]
denotes the greatest integer function, is
(b) one-one into function
(a) R (b) R Z
(c) many-one onto function
(c) (0, + ) (d) None of these
(d) one-one onto function
37. The function f : R R defined by 2x 1
44. Find the Range y
f (x) = (x – 1) (x – 2) (x – 3) is x5
53. |2x – 3| < |x + 5|, then x belongs to 59. The value of x, loge (x – 3) < 1 is
(a) (0, 1) b
(b) , 1 g 67.
(c) – 100 < x < 100
2
(d) – < x <
If f (x) = x – 3x + 1 and f (2) = 2f (), then is equal to
(c) (–1, 0) (d) (–1, 1)
1 1
61. Let f (x) l og x 2 25 and g(x) = logx 5 then f (x) = g(x) (a) (b)
2 2
holds for x belonging to
(a) R (b) (0, 1) (1, +) 1 1
(c) or (d) none of these
(c) (d) None of these 2 2
(a) – 2, –1
(b) – 2, 2 y=2x
(c) – 2, – 2 (d) None of these y=0 O X
(a) f (x) = x – |x| (b) f (x) = x + |x|
63. The domain of the function f (x) = log2 (log3 (log4 x)) is
(c) f (x) = 2x (d) None of these
(a) (–, 4) (b) (4, ) 69. The number of real solutions of the equation ex = x is
(c) (0, 4) (d) (1, ) (a) 1 (b) 2
64. The value of x, log½ x > log1/3 x is (c) 0 (d) none of these
(a) (0, 1] (b) (0, 1) 70. The number of real solutions of the equation log0.5 x = |x| is
(c) [0, 1) (d) none (a) 1 (b) 2
(c) 0 (d) none of these
SETS, RELATIONS & FUNCTIONS 35
(b) one - one but not onto 3 and f(x + y) = f(x) + f(y)+ xy, x, y R, then f (n) is
n 1
(c) onto but not one - one
equal to: (2017)
(d) neither one - one nor onot.
(a) 330 (b) 165
14. Let A and B be two sets containing four and two elements
(c) 190 (d) 255
respectively. Then the number of subsets of the set
A × B, each having at least three elements is: (2015) 19. Let S {x R : x 0 and 2 x 3 x
(a) 275 (b) 510
(c) 219 (d) 256
x 6 6 0} Then S : (2018)
15. In a certain town, 25% of the families own a phone and
15% own a car; 65% families own neither a phone nor a (a) Contain exactly four element
car and 2,000 families own both a car and a phone. (b) is an empty set.
Consider the following three statements : (c) contain exactly one element
(a) 5% families own both a car and a phone (d) contains exactly two elements.
(b) 35% families own either a car or a phone 20. Two sets A and B are as under :
(c) 40,000 families live in the town
A a, b R R : a 5 1 and b 5 1
Then, (2015/Online Set–1)
(a) Only (b) and (c) are correct
(b) Only (a) and (c) are correct
2 2
B a, b R R : 4 a 6 9 b 5 36 .
(c) All (a), (b) and (c) are correct Then : (2018)
(d) Only (a) and (b) are correct (a) neither A B nor B A
16. If f : R S, defined by (b) B A
21. Consider the following two binary relations on the set A 22. Let N denote the set of all natural numbers. Define two
={a, b, c} : binary relations on N as
R1={(c, a), (b, b), (a, c), (c, c), (b, c), (a, a)} and R1={( x, y) N × N : 2 x + y =10} and
R2={(a, b), (b, a), (c, c), (c, a), (a, a), (b, b), (a, c)}. Then : R2={( x, y) N N : x + 2 y =10}. Then :
(2018/Online Set–1) (2018/Online Set–3)
(a) both R1 and R2 are not symmetric. (a) Range of R1 is {2, 4, 8}.
(b) R1 is not symmetric but it is transitive. (b) Range of R2 is {1, 2, 3, 4}.
(c) R2 is symmetric but it is not transitive. (c) Both R1 and R2 are symmetric relations.
(d) both R1 and R2 are transitive. (d) Both R1 and R2 are transitive relations.
23. Let A, B and C be three events, which are pair-wise
independent and E denotes the complement of an event
(a) P A - P B (b) P A + P B
(c) P A - P B (d) P A + P B
38 SETS, RELATIONS & FUNCTIONS
9. Solution of |x – 1| > |x – 3| is
3
3. Solution set of, log3 (x2 – 2) < log3 | x | 1 is
2 (a) x < 2 (b) x > 2
(c) [1, 3] (d) none of these
(a) 2, 1
(b) 2, 2 10. Solution of |1/x – 2| < 4 is
(c) 2, 2 (d) none of these
(a) , 1 2
(b) 1 6 ,
1
11. Solution of x < 4 is
(a) 1,
(b) 0, log 2 3
4 x
(a) 2 3, 2 3 2 3, 2 3
(c) 1,
(d) 0, log 2 4 3 1,
(b) R 2 3, 2 3
5. Solution of the inequality x > 1 x is given by
(c) R 2 3, 2 3
(a) , 1 5 / 2 (d) none of these
12. The solution set of x2 + 2 < 3x < 2x2 – 5 is
(b)
5 1 / 2, (a) (b) [1, 2]
(c) (– , – 1) [5/2, ) (d) none
(c) , 1 5 / 2
5 1 / 2,
x 2 3x 4
13. The solution set of 1, x R is
x 1
(d)
5 1 / 2, 1
(a) 3, (b) 1, 1 3,
6. If x2 – 1 < 0 and x2 – x – 2 > 0, then x lies in the interval set
(a) (1, –1) (b) (–1, 1) (c) 1, 1 3, (d) none
(c) (1, 2) (d) {–1}
SETS, RELATIONS & FUNCTIONS 39
x2 1 1 | x | x 1
14. The number of integral solutions of is 21. Let f (x)
x2 1 2 [x] x 1
16. The solution set of log2 |4 – 5x| > 2 is 23. The largest set of real values of x for which
1
(a) 8 5 ,
(b) 4 , 8
5 5 f (x) ( x 2 )( 5 x )
2
x 4
is a real function is
17. Solution of 2x + 2|x| > 2 2 is 24. The domain of the function f (x) x 1 x 2 is
(a) , log 2 2 1
(a) 1,
1 1
, 1 (b) [–1, 1]
2 2
(b) 0,
1 1
(c) , ,
2 2
1
(c) , log 2
2
2 1
1
(d) , 1
2
(d) , log 2
2 1 ½,
25. The domain of the function f (x) = log10 log10 (1 + x3) is
18. The largest interval among the following for which (a) (–1, ) (b) (0, )
x12 – x9 + x4 – x + 1 > 0 is (c) [0, ) (d) (–1, 0)
(a) – 4 < x < 0 (b) 0 < x < 1
26. The domain of the function f (x) x 2 –[x]2 , where
(c) – 100 < x < 100 (d) – < x < [x] = the greatest integer less than or equal to x is
19. Solution of |x – 1| + |x – 2| + |x – 3| > 6 is (a) R (b) [0, +)
(c) (–, 0] (d) None of these
(a) [0, 4] (b) , 2 4,
27. The domain of f (x) l og x 2 1 (x) is
(c) , 0 4, (d) none
(a) ( 2, ) (b) (0, )
20. If f (x) = cos []x + cos [x], where [y] is the greatest integer
function of y then f (/2) is equal to (c) (1, +) (d) None of these
28. The domain of the real-valued function f (x) = loge |loge x | is
(a) cos 3 (b) 0
(a) (1, +) (b) (0, +)
(c) cos 4 (d) none of these
(c) (e, +) (d) None of these
40 SETS, RELATIONS & FUNCTIONS
29. If [.] denotes the greatest integer function then the domain
x2 , 0 < x < 2
of the real-valued function log x 1 2 x 2 x 2 is 35. Let f (x) = 2x – 3, 2 x < 3 . Then
x 2 , x 3
3 3
(a) , (b) , 2 (2, ) 3 3
2 2 (a) f f f f
2 2
1
(c) , 2 (2, ) (d) None of these
2 (b) 1 f f 5 5
f f
2 2
30. Let f (x) l og x 2 25 and g(x) = logx 5 then f (x) = g(x)
(c) f ( f (1)) = f (1) = 1
holds for x belonging to
(d) None of these
(a) R (b) (0, 1) (1, +) 36.
2 2 2
If a + b + c = 1, then ab + bc + ca lies in the interval
(c) (d) None of these
1 1
31. Let f : {x, y, z} {a, b, c} be a one-one function and only (a) , 1 (b) 0,
2 2
one of the conditions (i) f (x) b, (ii) f (y), = b
(iii) f (z) a is true then the function f is given by the set (c) [0, 1] (d) [1, 2]
2
(a) {(x, a), (y, b), (z, c)} (b) {(x, a), (y, c), (z, b)} 37. log2 (x – 3x + 18) < 4, then x belongs to
(a) (1, 2) (b) (2, 16)
(c) {(x, b), (y, a), (z, c)} (d) {(x, c), (y, b), (z, a)}
(c) (1, 16) (d) none of these
32. The function f : R R defined by f (x) = 6x + 6|x| is
38. If x = loga (bc), y = logb (ca) and z = logc (ab) then which of
(a) one-one and onto (b) many-one and onto the following is equal to 1?
(c) one-one and into (d) many-one and into (a) x + y + z (b) (1+ x)–1 + (1+ y)–1 + (1+ z)–1
33. A function whose graph is symmetrical about the y-axis is (c) xyz (d) none of these
given by
x x x 31
39. If 0 < x < 1000 and x , where [x] is
(a) f (x) l og e x x 2 1 2 3 5 30
the greatest integer less than or equal to x, the number of
(b) f (x + y) = f(x) + f (y) for all x, y R possible values of x is
2 f (n) 1
y=0 O X 41. If f (n + 1) , n = 1, 2, ... and f (1) = 2, then
2
f (101) equals
(a) f (x) = x – |x| (b) f (x) = x + |x| (a) 52 (b) 49
(c) f (x) = 2x (d) None of these (c) 48 (d) 51
SETS, RELATIONS & FUNCTIONS 41
1 1
42. The domain of function f (x) , where {x} 47. If < log0.1 x < 2 then
2
x 2 { x }2
1 1 1
(c) x does not lie between and
(c) (, ] (1, ) (d) none of these 100 10
2
1
43. If graph of y = f (x) is (d) the minimum value of x is
100
48. If f is an even function defined on the interval (–5, 5) then
y=x x 1
a value of x satisfying the equation f (x) = f is
x2
y = f (x)
–1 5 –3 5
(a) (b)
2 2
–1 – 5 –3 – 5
(c) (d)
2 2
Then f (x) can be 49. Let f (x) = [x] = the greatest integer less than or equal to x
(a) y = 2 e x
(b) y = 4 e x and g(x) = x – [x]. Then for any two real numbers x and y.
(a) f (x + y) = f (x) + f (y)
1
x 1 x (b) g(x + y) = g(x) + g(y)
(c) y e 2
(d) y e
4 (c) f (x + y) = f (x) + f (y + g(x))
44. The equation | |x – 1| + a | = 4 can have real solutions for x (d) none of these
if ‘a’ belongs to the interval 50. Let x N and let x be a perfect square. Let f (x) = the quotient
(a) (–, 4] (b) (–, –4] when x is divided by 5 and g(x) = the remainder when x is
(c) (4, + ) (d) [–4, 4] divided by 5. Then x = f (x) + g(x) holds for x equal to
45. If logk x. log5 k = logx 5, k 1, k > 0, then x is equal to (a) 0 (b) 16
(c) 25 (d) None of these
1 2
(a) k (b) 51. Let f (x) = [x] + [x + 1] – 3, where [x] = the greatest integer
5 < x. Then
(c) 5 (d) None of these (a) f (x) is a many- one and into function
(b) f (x) = 0 for inifinite number of values of x
46. If x 4 f (x) 1 sin 2x | f (x) | 2f (x), then f(-2) equals:
(c) f (x) = 0 for only two real values
(D) If both ASSERTION and REASON are false. 57. Assertion : Let A and B be two sets each with a
finite number of elements. Assume
(E) If ASSERTION is false but REASON is true.
that there is an injective mapping from
ax + b A to B and that there is an injective
54. Assertion : The function , (ad – bc0)
cx + d
mapping from B to A. Then there is a
bijective mapping from A to B.
a
cannot attain the value .
c Reason : An onto function is not necessarily
one-one.
Reason : The domain of the function
(a) A (b) B
b – dy
g(y) = is all the reals except a/c.
cy – a (c) C (d) D
(e) E
(a) A (b) B
58. Assertion : The domain of a function y = f (x) will
(c) C (d) D
be all reals if for every real x there
(e) E
exist y.
55. Assertion : The domain of the function
sin–1 x + cos–1 x + tan–1 x is [–1, 1] Reason : The range of a function y = f (x) will
be all reals if for every real y there exists
Reason : sin–1 x, cos–1 x are defined for |x| 1
and tan–1 x is defined for all x. a real x such that f (x) = y.
Using the following passage, solve Q.59 to Q.61 63. The number of solutions of equation f (x) = y2 where y is
a positive integer and x < 5 is
Passage –1
(a) 5 (b) 2
A rational function is defined as quotient of two
(c) 3 (d) None of these
polynomials, p(x) and q(x). The domain of the rational
function must be all reals except the roots of the equation 64. The number of solutions of f (x) = y2 where x > 5 must be
q(x) = 0. The range of rational function can be found by (a) 1 (b) 2
finding minimum and maximum values of the function. In (c) 3 (d) None of these
case p(x) and q(x) have a common factor x – . Then after Match the column
cancelling the common factor, the rational function must
assume a value at x = which should be deleted from the 65. Column–I Column–II
found range since is not there in the domain of the (A) f (x + y) = f (x) + f (y) (P) log3 x
rational function. (B) f (xy) = f (x) + f (y) (Q) tan–1 x
3x + 1 (C) f (x + y) = f (x) . f (y) (R) 3x
59. The range of the rational function f (x) = must be
2x + 1
x+y
(D) f (x) + f (y) = f (S) 3x
1 – xy
1 1
(a) R –
– (b) R – –
2 3 66. Column–I Column–II
(A) odd function (P) x – [x]
3
(c) R – (d) R
2
(B) even function
(Q) log x 1 x
2x + 1 1 x
60. The range of the rational function f (x) = (C) neither even nor odd (R) x log
2x 2 + 5x + 2
1– x
must be
(a) R – {0} (b) R – {–2} 2x / 2
(S)
1 2x / 2
2
(c) R – {0, –2} (d) R – 0,
3 Subjective
67. When 0 < x < 2 and [x] denotes greatest integer < x, then
2x 2 + 5x + 2 [sin x] + [cos x] + [sin x + cos x] takes exactly k integer
61. The range of the rational function f (x) =
2x + 1 values. Then k must be
must be
68. The range of the function x – 6 12 – x is an interval
(a) R – {0} (b) R – {–2}
of length 2 3 – k then k must be
2 3
(c) R – 0, – 2, (d) R –
3 2
1
69. If f x = x3 + x–3 then f (5) must be equal to
Using the following passage, solve Q.62 to Q.64 x
Passage –2 70. The least period of the function
Let f (x) be a function defined by
f(x) = 1! + 2! + 3! + 4! .... + x!, where x is a positive integer. cos (cos x) + sin (cos x ) + sin 4x is k
Answer the following questions :
62. The last digit of f (2007) will be then value of k must be
(a) 3 (b) 7
(c) 5 (d) 1
44 SETS, RELATIONS & FUNCTIONS
ANSWER KEY
EXERCISE - 1 : BASIC OBJECTIVE QUESTIONS
1. (b) 2. (c) 3. (c) 4. (c) 5. (d) 6. (b) 7. (c) 8. (a,c) 9. (c) 10. (d)
11. (a) 12. (b,d) 13. (b) 14. (a) 15. (a) 16. (a) 17. (c) 18. (b) 19. (a) 20. (a)
21. (d) 22. (c) 23. (a) 24. (d) 25. (a) 26. (b) 27. (i) (a); (ii) (d); (iii) (a); (iv) (c) 28. (b)
29. (c) 30. (d) 31. (d) 32. (b) 33. (d) 34. (c) 35. (c) 36. (d) 37. (b) 38. (b)
39. (a) 40. (a) 41. (d) 42. (a) 43. (b) 44. (a) 45. (c) 46. (a) 47. (b) 48. (b)
49. (d) 50. (a) 51. (c) 52. (c) 53. (c) 54. (i) (c) (ii) (a) (iii) (d) 55. (a) 56. (a)
57. (b) 58. (d) 59. (d) 60. (i) (a) (ii) (a) (iii) (a) 61. (b) 62. (d) 63. (b) 64. (a)
65. (a) 66. (d) 67. (c) 68. (b) 69. (c) 70. (a)
1. (a) (–5, 1) (3, ) ; (b) (– , 1] [2, 3) 2. (b) 3. (d) 4. (d) 5. (d) 6. (d) 7. (b)
8. (d) 9. (b) 10. (d) 11. (a) 12. (a) 13. (b) 14. (c) 15. (b) 16. (c) 17. (d)
18. (d) 19. (c) 20. (c) 21. (d) 22. (d) 23. (b) 24. (d) 25. (b) 26. (d) 27. (a)
28. (d) 29. (b) 30. (b) 31. (c) 32. (d) 33. (d) 34. (b) 35. (a,b,c) 36. (a) 37. (a)
38. (b) 39. (c) 40. (b) 41. (a) 42. (d) 43. (d) 44. (a) 45. (b,c) 46. (a) 47. (a,b,d)
48. (a,b,c,d) 49. (c) 50. (b,c) 51. (a,b) 52. (b,c) 53. (a,b) 54. (a) 55. (a) 56. (d) 57. (b)
58. (b) 59. (c) 60. (d) 61. (d) 62. (a) 63. (b) 64. (d)
65. (A) (R), (B) (P), (C) (S), (D) (Q) 66. (A) (Q), (B) (R, S), (C) (P)
Dream on !!
02
LIMITS
LIMITS 47
LIMITS
procedure.
Step–1 : Write down the given expression in the form of a tan x a
8. lim 1
x a x a
f x
rational function, i.e., , if it is not so.
g x 5. EXPONENTIAL & LOGARITHMIC LIMITS
Step–2 : If k is the highest power of x in numerator and To evaluate the exponential and logarithmic limits we use
denominator both, then divide each term in numerator the following results.
and denominator by xk.
a x 1
1 1. lim log e a
Step–3 : Use the result lim n 0 , where n > 0. x 0 x
x x
e x 1
3.4.1 Important Results : If m, n are positive integers and 2. lim 1
x 0 x
a 0 , b 0 0 are non-zero numbers, then
xn an n –1
4. lim n .a
a0
x a
x a
b , if mn
0
0, if mn 6. EXPONENTIAL LIMITS OF THE FORM
, if m n , a 0 b0 0
To evaluate the exponential limits of the form 1 we use the
, if m n , a 0 b0 0 following results.
If g (x) < f (x) < h (x) on an open interval containing ‘c’, and
(a) lim log f x log lim f x log ; (provided l > 0)
if : x a x a
g(x)
1
f(x) 7. If lim f x or – , then lim 0.
L x a x a f x
h(x)
9. LIMITS BY DE’L’ HOSPITAL’S RULE
x=c
If f (x) and g(x) be two functions of x such that
In words : If a function can be squeezed/sandwiched between
two functions whose limits at a particular point c have the 1. lim f x lim g x 0
x a x a
same value L, then that function must also have limit
L at x = c. 2. both are continuous at x = a,
Let f and g be two real functions with domain D. We define 4. f (x) and g(x) are continuous at the point x = a, then
four new functions f ± g, f g, f /g on domain D by setting
f x f ' x
( f ± g) (x) = f (x) ± g (x), ( f g) (x) = f (x) × g (x) lim lim , provided that g (a) 0.
x a g x x a g' x
( f /g)(x) = f (x)/g(x), if g(x) 0 for any x D.
Following are some results concerning the limits of these
The above rule is also applicale,
functions.
if lim f x and lim g x .
Let lim f x and lim g x m , if and m exist. x a x a
x a xa
is continuous at g (x) = m.
50 LIMIT, CONTINUITY & DIFFERENTIABILITY
SOLVED EXAMPLES
Example – 1 Example – 3
lim
x 1 x 3 2 1 2 3 1 Evaluate the following limits :
x 2 x 4 2 4 2.
sin cos x cos x
Example – 2 lim
x
sin x – cosec x .
2
a 2x 3x
Evaluate lim .
x a 3a x 2 x sin x
using lim 1
x 0 x
a 2x 3x
Sol. When x = a, assumes the indeterminate
3a x 2 x sin cos x cos x
Sol. xlim
sin x – cosec x
0 2
form .
0
sin cos x sin x cos x
a 2x 3x lim
Now, lim sin 2 x 1
x
x a 3a x 2 x 2
lim
a 2x 3x a 2x 3x 3a x 2 x sin co s x sin x
lim
x a
3a x 2 x 3a x 2 x a 2x 3x x
2
co s x
a x = –1.
lim
3a x 2 x
x a
a x a 2x 3x
lim
3a x 2 x 4 a 2
.
x a
a 2x 3x 2 3 3a 3 3
52 LIMIT, CONTINUITY & DIFFERENTIABILITY
Example – 5 Example – 7
tan 3 x – 3 tan x
Sol. Let L lim and x – t x 3– x
x / 3
cos x
3 Sol. L lim 3 + 3 – 12
3– x x/ 2
6 x 2 3 –3
27
tan 3 t – 3 tan t 3x + – 12
3 3 L lim 3x
L lim 27
t 0 x2
– 3x / 2
cos t
2 3x
32x – 12.3x 27
tan 3t 3tan 2 t – 1 L lim
L lim 3
x2
– 3x / 2 – 33
t 0 – sin t
(3x – 9) (3x – 3)
L – lim
tan 3t x2
3 x/2
–3 3 x
9 3.3x / 2
L lim .lim 3 tan 2 t – 1
t 0 – sin t t 0 3
(3x/2 + 3) (3x – 3)
tan 3t t L lim –
L – 3 lim
t 0 3t
lim
t 0 sin t
lim 3 tan 2 t – 1
t0
3
x 2
3x 3.3x / 2 9
L = –3 × 1 × 1 × 8 = – 24. – 6.6 – 36 – 4
L .
Example – 6 9 3.3 9 27 3
x a
e –e . x
lim a
tan
x a x–a 2a
Evaluate : lim 2 –
x a x
a x –1 [1 type of indeterminate form]
using lim l og a
x 0 x
a x
Sol. Since, lim 2 – 1 and lim .
x a x x a 2a
Sol. lim
e x
–e a
lim
e a
e x– a
–1
x a x –a x a x –a tan
x
a 2a
lim 2 – of the form (1)
x– a x a x
a e –1 x– a
e lim lim
x a x – a x a x – a x
tan
a 2a
Hence, lim 2 –
a x – a e a
x a x
e 1 lim
x a
x – a x a 2 a
LIMITS 53
a x
ex a
lim 2 – – 1 tan .
x 2a
L – 6 lim
cos3x 2 cos 2x 2 – x
x 0 4x –1
x –a x
lim tan .
x 2a
ex a
x
lim
h h
x 0
sin 2x 2 – x
lim tan
e h 0 a h 2a (putting, x – a = h)
1.1 1
L – 6. lim
– lim
h
h 0 a h
h
cot
2a
tan – cot
– 1 x 0 4x –1 cos 2x 2 – x
e
h / 2a lim 2a / h 1
– lim L – 6. –6
h 0 tan h / 2a h 0 a h – 1.1
e
0
( type of indeterminate form)
0 1 1
Sol. Let L = lim – [– form]
x 1 l n x x –1
sin 3x 2 0
Sol. Let lim [ form] Let us reduce it to an indeterminate form of the type 0/0.
x 0
l n cos 2x – x 2
0
x –1– l n x
Apply LH rule to get : L lim [0/0 form]
x 1 x –1 l n x
L lim
–6 x cos 3x 2 cos 2x 2 – x Apply LH rule to get,
x 0
4x –1 sin 2x 2
–x
1–1/x
L lim .
x 1 l n x +1 – 1 / x
– 6 lim
cos3x 2 cos 2x 2 – x
x 0 4x – 1
x –1
L lim .
x 1 x l n x + x –1
x
lim
x 0
sin 2x 2 – x Apply LH rule again,
Example – 11
lim f (x)
x 0
Evaluate the left hand and right hand limits of the function
defined by h | h |
lim f (0 h) lim
h 0 h 0 h
1 x 2 , if 0 x 1
f (x) at x = 1.
2 x, if x 1 hh 0
lim lim lim 0 0
h 0 h h 0 h h 0
(RHL of f(x) at x = 1)
1
lim f (x) lim f (1 h) Sol.(i) The graph of f(x) = is as shown in Fig. We observe that
x 1 h 0 x
as x approaches to 0 from the LHS i.e. x is negative and very
lim 2 (1 h) lim 1 h 1
h 0 h 0 1
close to zero, then the values of are negative and very
x
Clearly, lim f (x) lim f (x).
x 1 x 1 large in magnitude.
x | x |
, x0
If f (x) x show that xlim
0
f(x) does not
2 , x0
exist.
Sol. We have,
(LHL of f(x) at x = 0)
= xlim
0
f (x)
h | h |
lim f (0 h) lim
h 0 h 0 ( h) Similarly, when x approaches to 0 from the right i.e. x is
1
h h 2h positive and very close to 0, then the values of are very
lim lim lim 2 2 x
h 0 h h 0 h h 0
large and positive.
(RHL of f(x) at x = 0)
LIMITS 55
Example – 14
1 1 1
lim Thus we have, lim lim
x 0 x x 0 x x 0 x
mx 2 n, x 0
1
Hence, lim does not exist. If f (x) nx m,0 x 1
x0 x 3
nx m, x 1
1
(ii) The graph of f (x) = is shown in Fig.. We observe that
|x| For what values of integers m, n does the limits xlim
0
f(x)
as x approaches to 0 from LHS i.e. x is negative and close to
and lim f (x) exist.
0, then | x | is close to zero and is positive. Consequently, x 1
1
is large and positive. Sol. It is given that
|x|
lim f (x) and lim f (x) both exist
x 0 x 1
1
lim
x 0 | x | lim f (x) lim f (x) and lim f (x) lim f (x)
x 0 x 0 x 1 x 1
Also, if x approaches to 0 from RHS i.e. x is positive and
close to 0, then | x | is close to zero and is positive. lim f (0 h) lim f (0 h) and lim f (1 h) lim f (1 h)
h 0 h 0 h 0 h 0
1
Consequently, is large and positive. lim m( h) 2 n lim n (h) + m and
|x| h 0 h 0
n = m and n + m = n + m
m=n
Hence, xlim
0
f (x) and lim f(x) both sides for n = m.
x 1
Example – 15
| x | 1 , x 0
If f (x) 0 , x 0 . For what value (s) of a does
| x | 1 , x 0
Sol. We have,
1
lim
x 0 | x |
| x | 1 , x 0
f (x) 0 , x0
Thus, we have | x | 1 , x 0
1 1
lim lim
x 0 | x | x 0 | x |
x 1 , x 0
1 f (x) 0 , x0
Hence, xlim exists and it tends to infinity.. x 1 , x 0
0 |x|
56 LIMIT, CONTINUITY & DIFFERENTIABILITY
Example – 17
x, x 0
| x |
x, x 0 Evaluate the left hand and right hand limits of the function
lim f (x) lim f (0 h) lim h 1 1 Sol. The given function can be written as
x 0 x 0 h 0
= lim f (3 – h)
a bx , x 1 h 0
Suppose f (x) 4 , x 1
b ax , x 1 3– h –3
= lim
h 0 3 – h – 3
and if lim
x 1
f (x) = f (1). What are possible values of a and b ?
–h h
lim lim –1
h 0 – 3 h 0 –h
Sol. We have,
& R.H.L. = lim f (x)
lim f (x) f (1) x 3
x 1
= lim f (3 + h)
h 0
lim f (x) lim f (x) f (1)
x 1 x 1
3 h – 3
= lim
lim f (x) f (1) and lim f (x) f (1) h 0 3 h – 3
x 1 x 1
h h
lim f (1 h) 4 and lim f (1 h) 4 lim lim –1
h 0 h 0 h 0 h h 0 h
Hence left hand limit and right hand limit of f (x) at x = 3 are
lim {a b(1 h)} 4 and lim {b a(1 h)} 4 –1 and 1 respectively.
h 0 h 0
As left Hand Limit Right Hand Limit, Limiting value
a + b = 4 and b – a = 4 at x = 3.
Example – 18
– 2h
lim lim 2 2
h 0 – h h 0
x – x
, x 0
If f x x (R.H.L. of f (x) at x = 0)
2 , x 0
lim f (0 + h) lim f (0 + h)
h 0 h 0
(L.H.L. of f (x) at x = 0) 0
lim lim 0 0.
h 0 h h 0
lim f (0 – h)
h 0
–h– –h –h–h
lim lim
h 0 – h h 0 –h So, lim f (x) does not exist.
x 0
LIMITS 58
1 1 1
(a) (b) 2 1
2 (c) (d)
4a a b b ba
1
(c) 1 (d) 1
2 1 2 x 3
9. lim
x2 x2
x2 3
3. lim
x 3 x 2 3 3x 12 1 1
(a) (b)
8 3 3
1 2
(a) (b) (c) 8 3 (d)
5 5 3
Exponential & Logarithmic
3 3
(c) (d)
5 5 etan x e x
10. Limit
x 0 tan x x
1 x 2/3
4. lim (a) 1/2 (b) 0
x 1 1 x 1/3
(c) 1 (d) none
(a) 2 (b) 1
3x 1
12 11. lim is equal to
(c) (d) none of these x 0 x 1 1
3
(a) loge 9 (b) loge 3
5. lim
2 1 2x 3
is equal to (c) 0 (d) 1
x 1 2x 2 x 3
41/ n 1
12. lim is equal to
1 1 n 31/ n 1
(a) (b)
10 10 (a) log4 3 (b) 1
(c) 1 (d) None of these (c) log3 4 (d) none of these
Rationalization
3x 2 x
13. lim is equal to
2 x 3 x 0 4 x 3x
6. The value of lim is
x 7 x 2 49 (a) 1 (b) –1
(a) 2/9 (b) -2/49 (c) 0 (d) none of these
(c) 1/56 (d) -1/56
LIMITS 59
x 3 1 1 1 1
14. The value of l im l og a is 22. lim ... n equals
x 3 x 6 3
n 2 2 2 23 2
(a) 2 (b) -1
(a) loga6 (b) loga3
(c) 1 (d) 3
(c) loga2 (d) None of these
5n 1 3n 22n
x 2 ax b 23. Limit
e 1 n 5n 2n 32n 3
15. If , are the roots of x2 – ax + b = 0, then lim
x x (a) 5 (b) 3
is (c) 1 (d) zero
(a) – (b) –
1 1 1
log 5 ( ...to n terms)
(c) 2 (d) 2 24. lim (0.2) 4 8 16
is equal to
n
2x 1 (a) 2 (b) 4
lim 1
16. x 0 = (c) 8 (d) 0
1 x 2 1
x 1 3x 4 1 1
17. The value of xlim is equal to (c) (d)
x 2 x 8 6 3
Trigonometric Limit
(a) 2 (b) 3
(c) 1 (d) 0 2sin 2 3x
26. lim
40 5
x 0 x2
(2 x) (4 x)
18. lim (a) 6 (b) 9
x (2 x) 45
(c) 18 (d) 3
(a) –1 (b) 1
(c) 16 (d) 32 x
27. lim (1 x) tan
x 1 2
Limit
n + 2 ! n +1! ,
19. nN=
n n 3 ! (a) (b) + 2
2
(a) 0 (b) 1
2
(c) 2 (d) –1 (c) (d) none of these
1 2 n
20. l im 2
2
... is equal to x sin x
n
1 n 1 n 1 n 2 28. lim
x x cos 2 x
(a) 0 (b) – 1/2 (a) 0 (b)
(c) 1/2 (d) None of these (c) 1 (d) none of these
1 2 3 ... n
21. The value of lim is equal 1
n n 2 100 x 2 sin
lim x
29.
(a) (b) 1/2 x 0 sin x
(c) 2 (d) 0
(a) 1 (b) 0
1
(c) (d) none of these
2
60 LIMIT, CONTINUITY & DIFFERENTIABILITY
Limit
sin e x 2 1
38.
(a) (b) x 2 l n x 1
4 3
(c) (d) 0 (a) 0 (b) – 1
(c) 2 (d) 1
32. lim (1 cos x) cot 2 x
x 1
x x x x
39. The value of nlim cos cos cos ...cos n is
1 2 4 8 2
(a) –1 (b)
2
sinx
(c) 1 (d) none of these (a) 1 (b)
x
x
2 3 sin h cos h
6 6 (c) (d) None of these
33. lim sin x
h 0
3h 3 cosh sinh
(cos x 1) (cos x e x )
40. lim finite non-zero, then n is
(a) -2/3 (b) -3/4 x 0 xn
(c) -2 3 (d) 4/3 (a) 1 (b) 2
(c) 3 (d) 4
4 tan sin
34. lim is 1 Form
0
1 cos 2 2
41. lim (2 tan x)log tan x is equal to
x / 4
1
(a) (b) 1/2 (a) 0 (b) 1
2
(c) e (d) e–1
(c) 1 (d) 2
1/ x
cos ecx cot x 42. lim tan x is equal to
35. lim is equal to x 0
4
x 0 x
(a) 1 (b) e
1
(a) (b) 1 (c) e 2
(d) e–2
2
1/ x 2
1 1 5x 2
(c) (d) 1 43. lim is equal to
2 x 0 1 3x 2
cos sin x cosx (a) e (b) e1/2
36. The value of Limit 4
is equal to
x 0 x (c) e –2
(d) none of these
(a) 1/5 (b) 1/6 x2
x 1
(c) 1/4 (d) 1/2 44. lim is equal to
x x 1
(a) e (b) e–1
(c) e–2 (d) none of these
LIMITS 61
5/ x
45.
x 0
Limit 1 tan 2 x sin x
x 0
5 2 54. If f x x , where [.] denotes the greatest
(a) e (b) e 0
(c) e (d) none x 0
1/sin x
46. The limiting value of (cos x) as x 0 is
integer function, then xlim f x is equal to
(a) 1 (b) e 0
2x
1 cos 2x a b
1. lim is (2002) 7. If xlim 1 2 e 2 , then the values of a and b are
x 0 2x
x x
(2004)
(a) (b) –1
(a) a R, b R (b) a = 1, b R
(c) zero (d) does not exist
(c) a R, b = 2 (d) a = 1, b = 2
x
x 2 5x 3 8. Let and be the distinct roots of ax2 + bx + c = 0, then
2. lim 2 is equal to (2002)
x
x x2
1 cos (ax 2 bx c)
lim is equal to (2005)
(a) e4 (b) e2 x (x ) 2
(c) e3 (d) e
1 a2
x (a) ( ) 2 (b) ( ) 2
x 3 2 2
3. For x R, xlim
x 2
is equal to (2002)
a2
(a) e (b) e–1 (c) 0 (d) ( ) 2
2
(c) e–5 (d) e5
(a) 3 (b) 0
(c) 1 (d) 2
LIMITS 63
2x
1 1 a 4
(a) (b) 20. If lim 1 2 e3 , then ‘a’ is equal to:
4 2 x x x
(c) 1 (d) 2 (2016/Online Set–1)
sin ( cos 2 x) 3
14. lim is equal to : (2014) (a) 2 (b)
x 0 x2 2
2 1
(a) (b) (c) (d)
2 3 4
(c) 1 (d) –
(1 cos 2x)2
21. lim is : (2016/Online Set–2)
tan x 2 x2 (k 2) x 2 k x 0 2x tan x x tan 2x
15. If lim 5, then k is equal
x 2 x 2 4x 4
to: (2014/Online Set–2) 1
(a) – 2 (b)
2
(a) 0 (b) 1
(c) 2 (d) 3 1
(c) (d) 2
2
1 cos 2x 3 cos x
16. lim is equal to: (2015)
x 0 x tan 4x
cot x cos x
22. lim 3 equals : (2017)
1
x
2
2x
(a) 2 (b)
2
1 1
(c) 4 (d) 3 (a) (b)
24 16
2
e x cos x 1 1
17. lim is equal to : (2015/Online Set–1)
x 0 sin 2 x (c) (d)
8 4
(a) 2 (b) 3
3x 3
23. lim is equal to : (2017/Online Set–1)
5 3 x 3 2x 4 2
(c) (d)
4 2
1
x tan 2x 2x tan x (a) 3 (b)
18. lim is (2015/Online Set–2) 2
x 0
1 cos 2x 2
3 1
(a) 2 (b) -2 (c) (d)
2 2 2
(c) 1/2 (d) -1/2
64 LIMIT, CONTINUITY & DIFFERENTIABILITY
24. For each t R, let [t] be the greatest integer less than or 1
equal to t. Then
lim
27 + x - 3
3
26. x0
2 equals : (2018/Online Set–3)
1 2 15 9 - 27 + x
3
lim x ..... (2018)
x 0 x x
x
1 1
(a) does not exist (in R) (b) is equal to 0. (a) (b) -
3 3
(c) is equal to 15. (d) is equal to 120.
1 1
x tan2x - 2x tanx (c) - (d)
25. lim 2 equals: (2018/Online Set–2) 6 6
x0
1 - cos2x
1
(a) (b) 1
4
1 1
(c) (d) -
2 2
LIMITS 65
Limit
1 cos ax 2 bx c
2
(a) 1 (b) –1 x
x
(c) 2 (d) 0
1
3
(a) 0 (b) 2
2
Limit
4 1
x
2. x 0 x x2 a2 a2
sin l n 1 (c) 2 (d) 2
p 3 2 2
3
(a) 9 p (log 4) (b) 3 p (log 4) x 2 9x 20
3 2 9. Limit where [x] is the greatest integer not
(c) 12 p (log 4) (d) 27 p (log 4) x 5 x x
Limit
x 3
27 l n x 2
(c) b – a (d) – (a + b)
7.
x 3 x 2
9
(a) – 8 (b) 8
(c) 9 (d) – 9
66 LIMIT, CONTINUITY & DIFFERENTIABILITY
ax
13. The value of l im a 2 x 2 cot is 19. Limit n cos sin has the value equal to
x a 2 ax n 4n 4n
(a) /3 (b) /4
2a 2a
(a) (b) (c) /6 (d) none
4a a h 2 sin a h a 2 sin a
(c) (d) 4a 20. The value of l im
h 0 h
is
14. If [x] denotes the greatest integer x, then (a) 2a sin a + a2cos a (b) 2a sin a – a2 cos a
(c) 2a cos a + a2sin a (d) None of these
1
Limit
n
13 x 23 x ... n 3 x equals
n4 21. If f (a) = 2, f ' (a) = 1, g (a) = –1, g' (a) = 2, then
(a) 3 (b) 5
sin x
if x 0 (c) –3 (d) 0
15. If f x x where [x] denotes the
0
if x 0 sin 2 x + a sin x
22. If l im be finite, then the value of a and
x 0 x3
greatest integer less than or equal to x, then Limit f (x)
x 0 the limit are given by
equals (a) –2, 1 (b) –2, –1
(a) 1 (b) 0 (c) 2, 1 (d) 2, –1
(c) –1 (d) none 23. The value of
cos 2 x
tan x 2 2 2
, x 0 l im 11 / cos x 21 / cos x ... n1 / cos x is
16. If f (x) = x where x
2
0,
x 0
(a) 0 (b) n
[x] denotes the greatest integer less than or equal to x,
n n 1
then l im f x equals (c) (d)
x 0 2
(a) 1 (b) –1 n 1
1 1 2
(c) 0 (d) Does not exist 24. lim 1 e n e n ... e n
is equal to
n
n
(a) e (b) –e
2 3 sin h cos h
6 6 is equal to
17. l im (c) e – 1 (d) 1 – e
h 0
3 h 3 cos h sin h 25. The values of a, b and c such that
sin x
sin x – sin x 100 x 99sin x
26. lim equals 33. The value of lim , where [.]
x
1 – sin x + l n sin x x0
sin x x
2
represents greatest integral function is
(a) 1 (b) 2
(a) 199 (b) 198
(c) 3 (d) 4
(c) 0 (d) None of these
1/sin x
x2 / 2 34. The limiting value of (cos x) as x 0 is
e cos x
27. Limit
x 0 x 3 sin x (a) 1 (b) e
function, is equal to / x
a x bx cx
(a) 1 (b) 0 36. If lim , (a, b, c, > 0) is equal to
x 0
3
(c) e (d) Does not exists
(a) 1, if = 1 (b) abc, if = 1
x3 (c) abc, if = 3
2/3
(d) (abc) , if = 2
30. If Lim 1, a > 0, then a + 2b is equal to
x 0 a x bx – sin x
37. The value of a for which
(a) 36 (b) 37
4
(c) 38 (d) 40
lim
e –1
x
8, is
x0 x2 x2
4 4
sin x – x cos x x 4 20 sin 2 l og e 1
31. l im is equal to a 2
x0
4
x 4 e2x –1– 2x 4 (a) –2 (b) –1
(a) 0 (b) –1/6 (c) 1 (d) 2
(c) 1/6 (d) does not exist
aex – b cos x ce – x
38. If lim 2 , then
1 9 x 0 x sin x
32. If f (n + 1) = f n , n N and f (n) > 0 for all
2 f n
(a) a = 1 (b) b = 2
n N then l im f (n) is equal to (c) c = –2 (d) c = 0
n
–1 –5
(c) b (d) a
2 2
68 LIMIT, CONTINUITY & DIFFERENTIABILITY
40. If x is a real number in [0, 1] then the value of Using the following passage, solve Q.45 to Q.47
2m
lim lim [1 + cos (n!x)] is given by, where [x] Passage –2
m n
(c) C (d) D
1
(e) E x n – 1 x –1 n –1
2
(D) Lt (S) e
x 1 n x – 1
(T) 0
Subjective
sin x – tan x
51. The value of Lim is
x 0 tan –1 x – sin –1 x
LIMITS 70
12. The value of l im ((sin x)1/x + (1/x)sin x), where x > 0 is Multiple Answers Questions
x 0
x2
(2006) a a2 x2
17. Let L lim 4 , a 0 . If L is finite, them
(a) 0 (b) –1 x 0 x4
(c) 1 (d) 2
(2009)
1
(a) a = 2 (b) a = 1
13. If lim [1 x log (1 b 2 )] 2b sin 2 , b
x
> 0 and
x 0
1 1
(–, ], then the value of is (2011) (c) L (d) L
64 32
(a) (b) Let f (x)
1 x(1 |1 x |) 1
4 3 18. cos for x 1. then
|1 x | 1 x
(2017)
(c) (d)
6 2 (a) lim x 1 f (x) 0
1/ x 2
1 5x 2
23. lim ... . (1996)
x 0 1 3x 2
72 LIMIT, CONTINUITY & DIFFERENTIABILITY
24.
log (1 2h) 2 log (1 h)
(1997) (a h) 2 sin (a h) a 2 sin a
lim ... . 30. Evaluate lim (1980)
h 0 h2 h 0 h
25. For each positive integer n, let
a x 1
1 31. Use the formula lim log e a, to find
1 x 0 x
yn
n
n 1 n 2 ... n n .
n
26. If lim
x a
[f (x) g(x)] exists, then both lim
x a
f(x) and lim
x a
g(x)
33. The largest value of the non-negative integer a for which
exist. (1981)
Subjective Problems 1 x
ax sin x 1 a 1 x 1
x 1 lim is (2014)
x 1 x sin x 1 1 4
27. Evaluate the following limit lim 2 (1978)
x 1 2x 7x 5
ANSWER KEY
EXERCISE - 1 : BASIC OBJECTIVE QUESTIONS
1. (a) 2. (d) 3. (b) 4. (a) 5. (b) 6. (d) 7. (a) 8. (b) 9. (a) 10. (c)
11. (a) 12. (c) 13. (d) 14. (a) 15. (b) 16. (b) 17. (d) 18. (a) 19. (a) 20. (b)
21. (b) 22. (c) 23. (d) 24. (b) 25. (b) 26. (c) 27. (c) 28. (c) 29. (b) 30. (a)
31. (a) 32. (b) 33. (d) 34. (b) 35. (c) 36. (b) 37. (a) 38. (d) 39. (b) 40. (c)
41. (b) 42. (d) 43. (d) 44. (c) 45. (a) 46. (a) 47. (c) 48. (b) 49. (d) 50. (b)
51. (c) 52. (c) 53. (d) 54. (d) 55. (b) 56. (b) 57. (a) 58. (a) 59. (a) 60. (b)
11. (a) 12. (d) 13. (d) 14. (a) 15. (d) 16. (a) 17. (d) 18. (c) 19. (b) 20. (b)
21. (a) 22. (b) 23. (b) 24. (d) 25. (c) 26. (c)
1. (c) 2. (b) 3. (d) 4. (d) 5. (d) 6. (c) 7. (c) 8. (b) 9. (c) 10. (c)
2
11. (d) 12. (c) 13. (d) 14. (b) 15. (b) 16. (d) 17. (a,c) 18. (b,c) 19. 20. –1
1 1 2
21. h ( 2hr h 2 ), 22. e5 23. e2 24. –1 25. (1) 26. False 27. 28. 29. 0
128r 3
30. a2 cos a+ 2 a sin a 31. 2 n 2 32. e2 33. (2) 34. (0002) 35. (7)
Dream on !!
03
TRIGONOMETRY
75 TRIGONOMETRY
TRIGONOMETRY
Trigonometric Ratios of Standard Angles
TRIGONOMETRIC RATIOS & IDENTITIES
1. The meaning of Trigonometry T–Ratio Angle ()
0° 30° 45° 60° 90°
Tri Gon Metron
3 sides Measure
1 1 3
Hence, this particular branch in Mathematics was sin 0 1
2 2 2
developed in ancient past to measure 3 sides, 3 angles
and 6 elements of a triangle. In today’s time–trigonometric
functions are used in entirely different shapes. The 2 basic 3 1 1
cos 1 0
functions are sine and cosine of an angle in a right–angled 2 2 2
triangle and there are 4 other derived functions.
1
tan 0 1 3
3
H
P
1
cot 3 1 0
3
B
sin cos tan cot sec cosec 2
sec 1 2 2
3
P B P B H H
H H B P B P
2
cosec 2 2 1
3
2. Basic Trigonometric Identities
(a) sin2 + cos2 = 1 : –1 sin 1; –1 cos 1 R The sign of the trigonometric ratios in different quadrants
are as under :
(b) sec2 – tan2 = 1 : | sec| 1 R
3 3
sec cosec cos ec sec
tan cot cot tan 2 2
2 2
sin cos cos sin 3 3
2 2 tan cot cot tan
2 2
tan cot cot tan 3 3
2 2 sec cos ec cos ec sec
2 2
2 tan A 1 tan 2 A
(d) sin 2A = 2 ; cos 2A =
tan A tan B 1 tan A 1 tan 2 A
(f) tan (A B)
1 tan A tan B
(e) sin 3A = 3 sin A – 4 sin3 A
(f) cos 3 A = 4 cos3 A – 3 cos A
cot A cot B 1
(g) cot (A + B) = 3 tan A tan A
cot B cot A (g) tan 3A =
1 3tan 2 A
(h) sin2 A – sin2 B = cos2B – cos2A = sin (A + B) . sin (A – B) (a) 2 sin A cos B = sin (A + B) + sin (A – B)
CD CD A B C A B C
(d) cos C – cos D = – 2 sin sin (vii) cot cot cot cot .cot .cot
2 2 2 2 2 2 2 2
3 1 5 b
(b) sin 15º or sin = cos 75º or cos ; E a b 2 sin( ), where tan
12 2 2 12 a
a
3 1 5 E a 2 b 2 cos( ), where tan
cos 15º or cos = = sin 75º or sin ; b
12 2 2 12
Hence for any real value of , a 2 b 2 E a 2 b 2
n
3 1 sin
tan 75º = 2 3 = cot 15º = 2 sin ( n 1 )
3 1 2
sin
2
5 1
(c) sin or sin 18º = & (b) cos + cos ( + ) + cos ( + 2) + ...... + cos ( + n 1 )
10 4
n
sin
5 1 = 2 cos ( n 1 )
cos 36º or cos 2
5 4 sin
2
TRIGONOMETRY 79
(d) y = cot x,
12. Graphs of Trigonometric Functions
x R – {n; n z}; y R
(a) y = sin x,
x R ; y [–1, 1]
y
x
0
(e) y = cosec x,
x R – {n; n Z}; y ––][1, )
(b) y = cos x,
x R ; y [–1, 1]
(f) y = sec x,
x R 2n 1 ; n Z ; y ––][1, )
(c) y = tan x, 2
x R 2n 1 ; n Z ; y R
2
80 TRIGONOMETRY
SOLVED EXAMPLES
Example – 1
Multiplying and dividing by 2 sin , we get
Solve : If sec and cosec are the roots of 2
x2 – px + q = 0, then show p2 = q (q + 2).
2 cos .sin
2
Sol. Since, sec and cosec are roots of x – px + q = 0
cot 2 2 sin sin
sec + cosec = p and sec . cosec = q cos cos
2 sin sin
2 2
1
sin + cos = p sin . cos and sin . cos
q sin sin
cot .
cos cos
p
sin + cos = .
q Example – 3
Squaring both sides, we get Solve : Prove that
p2 tan A + 2 tan 2A + 4 tan 4A + 8 cot 8A = cot A.
sin2 + cos2 + 2 sin . cos
q2
1 tan 2 4A
p 2 Sol. L.H.S. = tan A + 2 tan 2A + 4 tan 4A + 8
1 + 2 sin . cos 2 tan 4A
q2
4 tan 2 4A 4 4 tan 2 4A
2 p2 = tan A + 2 tan 2A +
or 1 2 p2 = q (q + 2). tan 4A
q q
= cot A = R.H.S.
Note: Students are adviced to learn above result as formulae.
cos
cot 2 i.e., tan A + 2 cot 2A = cot A
sin
2
TRIGONOMETRY
82 82
TRIGONOMETRY
Example – 4 Example – 6
sin12.sin 36 3 3
1 LHS = cos 3A + {cos A + 2 cos (180°–A) cos 60°}
. 4 4
64 sin 12.sin 36 64
3 3 1 3
Example – 5 LHS= cos3A cos A 2cos A cos3A=RHS
4 4 2 4
Solve : Prove that : ALITER
tan A + tan (60° + A) – tan (60° – A) = 3 tan 3A We have,
cos A + cos (120° + A) + cos (240° + A)
Sol. We have, = cos A + 2 cos (180° + A) cos 120°
LHS = tan A + tan (60° + A) – tan (60° – A)
CD C D
cos C cos D 2cos 2 cos 2
3 tan A 3 tan A
LHS tan A
1 3 tan A 1 3 tan A = cos A – cos A = 0
cos3 A + cos3 (120° + A) + cos3 (240° + A)
8tan A
LHS tan A = 3 cos A cos (120° + A) cos (240° + A)
1 3tan 2 A
[ a + b + c = 0 a3 + b3 + c3 = 3abc]
9 tan A 3tan 3 A = 3 cos A cos (180° – 60° + A) cos (180° + 60° + A)
LHS
1 3tan 2 A = 3 cos A cos {180° – (60° – A)} cos {180° + (60° + A)}
= 3 cos A cos (60° – A) cos (60° + A)
3tan A tan 3 A
LHS 3 = 3 tan 3A = RHS 1 3
2 3 cos3A cos3A
1 3tan A 4 4
TRIGONOMETRY 83
Example – 7
(cos 60 sin18)(cos36 sin 30)
LHS
3 3
Solve : Prove that : sin 3A sin A + cos 3A cos A = cos 2A 3 (cos 60 sin18)(cos36 sin 30)
Sol. We have,
1 5 1 5 1 1
cos3A 3cos A 2 4 4 2
cos3 A LHS
4 1 5 1 5 1 1
2 4 4 2
LHS = sin 3A sin3 A + cos 3A cos3 A
1
LHS = {3 (cos A cos 3A + sin A sin 3A) + (cos2 3A–sin2 3A)} Example – 9
4
Sol. We have,
1
LHS = {3 cos 2A + cos 3 (2A)} 16 sin2 27° = 8 (1 – cos 54°)
4
16 sin2 27° = 8 (1 – sin 36°)
1
LHS = {3 cos 2A + (4 cos3 2A – 3 cos 2A)}
4 10 2 5
16sin 2 27 8 1
= cos3 2A = RHS 4
Example – 8
Solve : Prove that : tan 6° tan 42° tan 66° tan 78° = 1
16sin 2 27 2 4 10 2 5
Sol. We have,
16sin 2 27 8 2 10 2 5
sin 6 sin 42 sin 66 sin 78
LHS
cos 6 cos 42 cos 66 cos 78 16 sin2 27° = (5 5) (3 5) 2 (5 5) (3 5)
2 2
LHS
(2sin 66 sin 6)(2sin 78 sin 42)
(2cos66 cos6)(2cos78 cos 42)
16 sin2 27° = 5 5 3 5 2 (5 5) (3 5)
LHS
(cos 60 cos 72)(cos36 cos120) 16 sin2 27° = 5 5 3 5
(cos60 cos 72)(cos36 cos120)
sin 27 5 5 3 5
84 TRIGONOMETRY
Example – 10 Example – 11
2 1 cos 2B 1 cos 2C
1 = sin A
y or, y 3 2 2
3
1
y does not lie between 1/3 and 3. = [2 + 2 sin2 A – (cos 2B + cos 2C)]
2
TRIGONOMETRY 85
(iv) L.H.S.
1 2B 2C
= [2 + 2 sin2 A – 2 cos
2 2 A B C
cos cos cos
2 2 2
2B 2C
cos ]
2
B C B C
= 1 + sin2 A – cos (B + C) . cos (B – C) A 22 22
sin 2cos cos
2 2 2
= 2 – cos2 A + cosA . cos (B – C)
= 2 + cos A [–cos A + cos (B – C)]
= 2 + cos A [cos (B + C) + cos (B – C)]
= 2 + cos A × 2 cos B . cos C A A BC BC
2sin cos 2cos cos
= 2 + 2 cos A . cos B . cos C 4 4 4 4
(iii) L.H.S.
A A A BC
2sin cos 2cos cos
A B C 4 4 4 4
cos 2 cos2 cos 2
2
2
2
A
A 1 cos B 1 cos C sin 4
cos2 A
2 2 2 2cos
4 B C
cos
4
A 1
1 cos 2 (cos B cos C)
2 2
BC
A 1 BC
1 cos 2 2cos
BC sin 4
.cos A
2 2 2 2 2cos
4 B C
cos
A A BC 4
1 1 sin 2 sin .cos
2 2 2
A A B C B C
2 sin sin cos cos
2
2 2 A 4 4
2cos
4 C B
cos
A B C B C 4 4
2 sin cos cos
2
2 2
AC AC 2AC
cos x 0, x 2 n, n Z,
2cos .cos 2 cos
2 2 2 cos 2x 1 , 2x 2k, i.e. x k, k Z.
2
AC AC 2AC
3 6
2cos .cos 2cos
2 2 2
Answer : n, k (n, k Z)
A C A C A C 2 6
2cos cos cos
2 2 2
A C A C A C Solution of Equations Reducible to
2cos cos cos
2 2 2
Quadratic Equations
1
Answer : ± cos–1 + 2n (n Z).
3
TRIGONOMETRY 87
Example – 15
Equations of the form
Example – 17 Example – 19
x n, n Z, Example – 20
sin x 0,
3x k, x k, k Z.
cos3x 0, 2 6 3 x x
Solve : 2 sin cos2 x – 2 sin sin2 x = cos2 x – sin2 x.
2 2
Answer : k (n, k Z).
6 3
Sol. On the left-hand side of the equation we put the factor
Solving Equ. with the Use of x
2 2
2 sin before the parentheses :
cos = sin = for Lowering a Degree 2
Example – 18 x
2 sin (cos2 x – sin2 x) = cos2 x – sin2 x.
2
Solve : sin2 x + sin2 2x = 1
Replacing the expression cos2 x – sin2 x by cos 2x
according to formula (2), we get
1– cos 2x 1– cos 4x
Sol. 1 cos 2x + cos 4 x = 0 2 cos
2 2 x
2 sin cos 2x = cos 2x,
3 x cos x = 0. 2
The last equation is equivalent to the collection of two
equations. x
or 2 sin cos 2x – cos 2x = 0
2
(a) cos 3 x = 0, 3 x = + n, x = n, n Z
2 6 3 x
cos 2x 2 sin – 1 = 0
(b) cos x = 0, x = + k, k Z 2
2
The set of solutions of equation (b) is a subset of the set
of solutions of (a) and, therefore, in the answer we write cos 2x 0, x 4 2 n, n Z,
only roots equation (a).
sin x 1 , x –1k 2k, k Z.
n 2 2
Answer : (n Z). 3
6 3
Answer : n, (–1)k + 2k (n, k Z).
4 2 3
TRIGONOMETRY 89
(a) Equations of the form P (sin x ± cos x, sin x cos x) = 0, Solve : 3 cos x + 4 sin x = 5.
where P (y, z) is a polynomial, can be solved by the
change.
x x
cos x ± sin x = t 1 ± 2 sin x cos x = t2. 1– tan 2 2 tan
Sol. 3 2 4 2 5,
Let us consider an example. 2 x 2 x
1 tan 1 tan
Example – 21 2 2
1 f (x) = (x)
cos cos x + sin sin x = ,
4 4 2
Example – 23
2
cos x –
4 2 Solve : 1 – cos x sin x, x
x – 2 n, n Z,
4 4 1– cos 0,
Sol.
sin x 0.
x 2 n, n Z.
4 4 Under the condition that both sides of the equation are
nonnegative, we square them:
Answer : + 2 n, 2 n (n Z). 1 – cos x = sin2 x, 1 – cos x = 1 – cos2 x,
2
cos2 x – cos x = 0, cos x (cos x –1) = 0.
(b) Equations of the form a sin x + b cos x + d = 0, where a,
b, and d are real numbers, and a, b 0, can be solved by
(1) cos x = 0, x = + n, n Z,
the change. 2
(2) cos x = 1, x = 2k, k Z. But since sin x 0
x x
1– tan 2 2 tan 5
cos x 2 , sin x 2 , and x we leave x = 2, .
2 x 2 x
2
1 tan 1 tan
2 2
Answer : 2, 5 .
x + 2n (n Z) 2
90 TRIGONOMETRY
x x 1
Solve : cos – 2sin x sin x 1 sin – 2cos x cos x 0. sin x cos y ,
4 4 Solve : 4
3tan x tan y.
x x
Sol. cos sin x –2 sin2 x + cos x + sin cos x –2 cos2 x = 0.
4 4 Sol. We transform the second equation & get
3 sin x cos y – sin y cos x = 0.
x 5x Substituting now the value of the product, sin x cos y
sin x + cos x – 2 (sin2 x + cos2 x) = 0, sin + cos x = 2.
4 4 from the first equation into the equation obtained, we
get a system.
5x
Since the functions sin and cos x have the greatest 3
4
cos x sin y 4
5x .......(1)
value equal to 1, their sum is equal to 2 if sin =1 and sin x cos y 1
4 4
cos x = 1 simultaneously, i.e.
Adding together the equations of system (1) and then
subtracting the first equation from the second, we get a
5x 5x
sin 1, 2n, system which is equivalent to system :
4 4 2
cos x 1, x 2k n, k Z ; sin x y 1,
1 .......(2)
2 8 1 4n sin x – y ,
2k n, k 2
5 5 5
whence we have
Since k Z, it follows that n = 1 + 5m (m Z), and then
x = 2 + 8m, m Z
x y 2 2k,
Answer : 2 + 8m, m Z
x – y – 2l .......(3)
6
and
x y 2 2k,
.......(4)
x – y – 5 2l
6
527 427
(a) (b) 1
7200 7200 (a) (b)
(c) 1 – (d) 1 +
(c) (d)
7200 427
8. tan x is defined for all x in
2. The degree and radian measure of the angle between the
(a) R (b) R – {n: n I}
hour-hand and the minute-hand of a clock at twenty
minutes past seven is
(c) R {(2n 1) : n I} (d) none of these
2
5 5
(a) (b)
4 8 9. cot x is defined for all x in
(a) R (b) R – {n: n I}
5 5 c
(c) (d)
7 9
(c) R {(2n 1) : n I} (d) none of these
3. If the perimeter of a sector of a circle, of area 2
25 sq. cms. is 20 cms then area of a sector is
10. Which of the following is not correct ?
(a) 20 sq. cms (b) 24 sq. cms
(c) 50 sq. cms (d) 25 sq. cms 1
(a) sin (b) cos = 1
5
3
4. Number of sides of regular polygon of interior angle
4 1
(c) sec (d) tan = 20
is 2
(a) 10 (b) 5 11. If sin and cos are the roots of the equation
(c) 8 (d) 9 ax2 – bx + c = 0, then a, b and c satisfy the relation :
5. The sum of two angles is 5 and their difference is 60°. (a) a2 + b2 + 2 ac = 0 (b) a2 – b2 – 2 ac = 0
Then the angles are (c) a2 + c2 – 2 ab = 0 (d) a2 – b2 + 2 ac = 0
(a) 480°, 420° (b) 470°, 450° 12. If x R and x 0, then which of the following is not
(c) 520°, 580° (d) 360°, 120° possible ?
Trigonometric Ratios
1 1
6. cos 24º + cos 5º + cos 175º + cos 204º + cos 300º = (a) 2sin x (b) 2cos x
x x
(a) 1/2 (b) –1/2
1 1
3 (c) 2sin x (d) sin x
(c) (d) 1 x x
2
TRIGONOMETRY
92 92
TRIGONOMETRY
13. In a trangle ABC, if cotA cotB cotC > 0, then the triangle is 20. If 3 sin – 5 cos = a, then 5 sin + 3 cos is equal to
(a) acute angled (b) right angled
(a) 1 – a (b) 1 a 2
(c) obtuse angled (d) does not exist
14. Which of the following is correct – (c) 34 a 2 (d) 34 a 2 or 34 a 2
(a) sin 1º > sin 1 (b) sin 1º < sin 1
3
21. The value of tan tan is
8 8
(c) sin 1º = sin 1 (d) sin1º sin1
180
(a) 0 (b) 1
15. If sin (x –y) = cos (x + y) = 1/2, then the values of x and y
lying between 0º and 180º are given by 1
(c) (d) none of these
2
(a) x = 45º, y = 15º (b) x = 45º, y = 135º
(c) x = 165º, y = 15º (d) none of these 22. If tan = – 4/3, then sin is
sec tan 4 4 4 4
16. If 5 sin = 3, then is equal to (a) but not (b) or
sec tan 5 5 5 5
4 4
1 (c) but not (d) none of these
(a) (b) 4 5 5
4
23. The value of cos 1º cos 2º cos 3º .... cos 179º is
(c) 2 (d) none of these
(a) 1/ 2 (b) 0
17. A value of satisfying cos + 3 sin = 2 is
(c) 1 (d) None of these
5 4
(a) (b) 24. If sinx + sin2x = 1, then the value of
3 3
cos 12x + 3cos10x + 3cos8x + cos6x – 1 is equal to
2 (a) 0 (b) 1
(c) (d)
3 3 (c) –1 (d) 2
18. Which of the following is correct ? Trigonometric Identities
(a) cos 1 > cos 2 (b) cos 1 < cos 2
1 tan 2 15
(c) cos 1 = cos 2 (d) none of these 25. The value of is
1 tan 2 15
1 sin A sin A 1
19. If , for all permissible
1 sin A cos A cos A 3
(a) (b) 1
2
values of A, then A may belongs to
(a) First Quadrant (b) Second Quadrant 1
(c) (d) 3
(c) Third Quadrant (d) Fourth Quadrant 2
TRIGONOMETRY 93
17 5 3
26. If sec A = and cosec B = ; if A, B lies in first quadrant 29. If f (x) = 3 sin 4 x sin 4 3 x 2
8 4 2
85 85 (a) – 1
(c) (d)
84 84 (b) 0
(c) 1
27. The two legs of a right triangle are
(d) not a constant function
3 3 30. The sines of two angles of a triangle are equal to
sin + sin and cos – cos . The
2 2
5 99
& . The cosine of the third angle can be :
length of its hypotenuse is 13 101
28. Which of the following when simplified reduces to unity ? 245 255
(a) (b)
1313 1313
2 (c) x 2 3 1 a x a 0
(c)
1
1 tan 2
2 2 2
4 sin cos 4 tan
(d) 3x 1 a x a 3 0
1 sin 2
(d) 2 32. If = and = , then tan equals
sin cos 2
3
35. If < 2 < , then 2 2 2cos 4 is equal to (c) p 2 p
2
(d) none of these
2
(a) –2cos (b) –2sin 42. If cos 2x + 2 cos x = 1 then sin2x (2–cos2x) is equal to
(c) 2cos (d) 2sin (a) 1 (b) –1
(c) 5 (d)
36. If A – B = , then (1 + tan A) (1 – tan B) = 5
4
43. If A + C = B, then tan A tan B tan C is
(a) 1 (b) 2
(a) tan A tan B + tan C
(c) –1 (d) –2
(b) tan B – tan C – tan A
37. If tan and tan are the roots of the equation (c) tan A + tan C – tan B
2 2
8x2 – 26x + 15 = 0 then cos ( + ) is equal to (d) –(tan A tan B + tan C)
627 627 1 1
(a) (b) 44. If x + = 2 cos , then x3 + 3 =
725 725 x x
1 1
38. If sin + sin = a and cos – cos = b then tan is
2 (c) cos 3 (d) cos 3
2 3
equal to
45. If tan, tan are the roots of the equation
a b x2 + px + q = 0 (p 0), then
(a) (b)
b a
(a) sin () = –p (b) tan () = p/ (q – 1)
(c) a 2 b2 (d) none of these (c) cos () = 1 – q (d) none of these
TRIGONOMETRY 95
cos 12° . cos 24° . cos 36° . cos 48° . cos 72° . cos 84° is (b) tan A tan 2 A tan 3 A
(c) tan A tan 2A – tan 2 A tan 3 A – tan 3 A tan A
1 1
(a) (b) (d) none of these
64 32
3 5 7
55. cos cos cos cos is equal to
1 1 8 8 8 8
(c) (d)
16 128
49. The value of sin 78° – sin 66° – sin 42° + sin 6° is 1 2
(a) 1/2 (b)
2 2
1 1
(a) (b)
2 2 1 2
(c) 1/8 (d)
(c) –1 (d) none of these 2 2
1
(c) a = 3, b = –1 (d) (c) 2 (d)
3 2
96 TRIGONOMETRY
(a) 0 (b) 1
(c) (2n + 1) ; n I (d) None of these
2
1 1
(c) (d) 67. In 2 cos2 + 3sin = 0, then the general value of is–
2 4
(a) 4 (b) 0
(c) 2n± – ; n I
(c) 5 (d) None of these 4 6
1. The number of solutions of tan x + sec x = 2 cos x in 8. In a triangle ABC, a = 4, b = 3, A = 60°, then c is the root
[0, 2) is (2002) of the equation. (2002)
(a) 2 (b) 3 (a) c2 – 3c – 7 = 0 (b) c2 + 3c + 7 = 0
(c) 0 (d) 1 (c) c2 – 3c + 7 = 0 (d) c2 + 3c – 7 = 0
ABC A 5 C 2
2. In a triangle ABC, 2ca sin is equal to 9. In a ABC, tan , tan , then (2002)
2 2 6 2 5
(2002) (a) a, c, b are in AP (b) a, b, c are in AP
(a) a2 + b2 – c2 (b) c2 + a2 – b2 (c) b, a c are in AP (d) a, b, c are in GP
(c) b2 – c2 – a2 (d) c2 – a2 – b2
10. The equation a sin x + b cos x = c where | c | a 2 b 2 has
4xy (2002)
3. sin 2 is true, if and only if (2002)
(x y)2 (a) a unique solution
(a) x – y 0 (b) x = – y (b) infinite number of solutions
(c) x + y 0 (d) x 0, y 0 (c) no solution
(d) None of the above
1 tan 2 15
4. The value of is (2002)
1 tan 2 15 11. If is a root of 25 cos2+ 5 cos – 12 = 0, , then
2
(a) 1 (b) 3 sin 2is equal to (2002)
3 24 24
(c) (d) 2 (a) (b)
2 25 25
4 13 13
5. If tan , then sin is (2002) (c) (d)
3 18 18
12. The sum of the radii of inscribed and circumscribed circles
4 4 4 4 for an n sided regular polygon of side a, is (2003)
(a) but not (b) or
5 5 5 5
a
4 4 (a) a cot (b) cot
(c) but not (d) None of these n 2 2n
5 5
a
1 (c) a cot (d) cot
6. If sin ( + ) = 1, sin ( – ) , then tan 2n 4 2n
2
(+ 2) tan (2+ ) is equal to 13. If in a triangle ABC
(2002)
C A 3b
(a) 1 (b) –1 a cos 2 c cos 2 ,
2 2 2
(c) zero (d) None of these
2 2
then the sides a, b and c (2003)
7. If y = sin + cosec , 0, then (2002)
(a) are in AP (b) are in GP
(a) y = 0 (b) y 2
(c) are in HP (d) satisfy a + b = c
(c) y – 2 (d) y 2
TRIGONOMETRY
98 98
TRIGONOMETRY
value of cos
is (2004) (a) 1 7 (b)
4 7
2 4 3
6 3
(a)
65
(b)
130 (c)
4 7 (d)
1 7
3 4
3 6 23. A tower stands at the centre of a circular park. A and B are
(c) (d)
130 65 two points on the boundary of the park such that AB (=a)
subtends an angle of 60° at the foot of the tower and the
17. The sides of triangle are sin , cos and 1 sin cos angle of elevation of the top of the tower from A or B is 30°.
The height of the tower is (2007)
for some 0 . Then the greatest angle of the triangle
2 2a
is (2004) (a) (b) 2a 3
3
(a) 120° (b) 90°
(c) 60° (d) 150° a
(c) (d) 3
18. A person standing on the bank of river observes that the 3
angle of elevation of the top of a tree on the opposite bank
of the river is 60° and when he retires 40 meters away from
the tree the angle of elevation becomes 30°. The breadth of
the river is (2004)
(a) 40 m (b) 30 m
(c) 20 m (d) 60 m
TRIGONOMETRY 99
24. AB is a vertical pole with B at the ground level and A at the 28. If A = sin2 x + cos4 x, then for all real x (2011)
top. A man finds that the angle of elevation of the point A
from a certain point C on the ground is 60°. He moves away 13
(a) A 1 (b) 1 A 2
from the pole along the line BC to a point D such that 16
CD = 7 m. From D the angle of elevation of the point A is
45°. Then the height of the pole is (2008) 3 13 3
(c) A (d) A 1
4 16 4
7 3 1 7 3 1
(a) m (b) m 29. The possible values of (0, ) such that
2 3 1 2 3 1 sin () + sin (4) + sin (7) = 0 are (2011)
7 3 7 3 2 4 3 8 5 2 3 8
(c) ( 3 1) m (d) ( 3 1) m (a) , , , , , (b) , , , , ,
2 2 9 4 9 2 4 9 4 12 2 3 4 9
r
1 33. Let fk (x) 1 (sin kx + cos kx) where x R and
(b) there is a regular polygon with k
R 2
k 1. Then f4(x) – f6(x) equals : (2014)
r 2
(c) there is a regular polygon with 1 1
R 3 (a) (b)
12 6
r 3 1 1
(d) there is a regular polygon with
R 2 (c) (d)
3 4
100 TRIGONOMETRY
34. A bird is sitting on the top of a vertical pole 20 m high and 40. If 0 x 2 , then the number of real values of x,
its elevation from a point O on the ground is 45°. It flies off which satisfy the equation
horizontally straight away from the point O. After one
second, the elevation of the bird from O is reduced to 30°. cosx + cos2x + cos3x + cos4x = 0, is : (2016)
Then the speed (in m/s) of the bird is : (2014) (a) 5 (b) 7
(c) 9 (d) 3
(a) 20 ( 3 1) (b) 40 ( 2 1)
41. The number of x [0, 2] for which
(c) 40 ( 3 2) (d) 20 2
2 sin 4 x 18 cos2 x 2 cos4 x 18 sin 2 x = 1 is
35. The number of values of in [0, 2] for which : (2016/Online Set–1)
2 sin3 - 7 sin2 + 7 sin = 2, is:(2014/Online Set–1) (a) 2 (b) 4
(a) 6 (b) 4 (c) 6 (d) 8
(c) 3 (d) 1
42. If A > 0, B > 0 and A + B = , then the minimum value of
6
36. If 2 cos + sin 1 then 7 cos + sin is
2 tanA + tanB is : (2016/Online Set–2)
equal to (2014/Online Set–2)
(a) 3 2 (b) 2 3
1
(a) (b) 2 2
2 (c) 4 2 3 (d)
3
11 46
(c) (d) 43. If 5(tan2 x – cos2 x) = 2cos 2x + 9, then the value of cos 4x
2 5 is (2017)
37. If the angles of elevation of the top of a tower from three
collinear points A, B and C on a line leading to the foot of 3 1
(a) (b)
the tower, are 300, 450 and 600 respectively, then the ratio, 5 3
AB : BC, is: (2015)
2
(a) 1: 3 (b) 2 : 3 (c) (d)
9 9
(c) 3 :1 (d) 3: 2 44. Let a vertical tower AB have its end A on the level ground.
Let C be the mid-point of AB and P be a point on the
a ground such that AP = 2AB. If BPC = , then tan is
38. In a ABC, 2 3 and C 60º . Then the
b equal to: (2017)
ordered pair (A, B) is equal to : (2015/Online Set–1) 6 1
(a) (45º, 75º) (b) (75º, 45º) (a) (b)
7 4
(c) (105º, 15º) (d) (15º, 105º)
2 4
3 1 (c) (d)
39. If cos + cos and sin + sin and is the 9 9
2 2
45. If sum of all the solutions of the equation
arithmetic mean of and , then sin 2 + cos 2 is
1
equal to: (2015/Online Set–2) 8cos x. cos x .cos x 1 in 0, is k,
6 6 2
3 7 then k is equal to : (2018)
(a) (b)
5 5
20 2
4 8 (a) (b)
9 3
(c) (d)
5 5
13 8
(c) (d)
9 9
TRIGONOMETRY 101
46. If tanA and tanB are the roots of the quadratic equation, 49. A tower T1 of height 60m is located exactly opposite to a
3x 2 - 10x - 25 = 0, then the value of tower T2 of height 80m on a straight road. From the top of
3 sin 2 (A + B) –10sin(A + B).cos(A + B) –25 T1, if the angle of depression of the foot of T2 is twice the
cos2(A +B) is : (2018/Online Set–1) angle of elevation of the top of T2, then the width (in m) of
(a) -10 (b) 10 the road between the feet of the towers T1 and T2 is :
(c) -25 (d) 25 (2018/Online Set–2)
47. An aeroplane flying at a constant speed, parallel to the (a) 10 2 (b) 10 3
horizontal ground, 3 km above it, is observed at an
0
(c) 20 3 (d) 20 2
elevation of 60 from a point on the ground. If, after five
seconds, its elevation from the same point, is 300, then the 50. If an angle A of a ABC satisfies
speed (in km/hr) of the aeroplane, is :
5 cos A + 3 = 0, then the roots of the quadratic equation,
(2018/Online Set–1)
9x2 + 27x + 20 = 0 are : (2018/Online Set–3)
(a) 1500 (b) 1440
(a) sec A, cot A (b) sin A, sec A
(c) 750 (d) 720
(c) sec A, tan A (d) tan A, cos A
48. The number of solutions of sin 3x = cos 2x, in the interval
51. A man on the top of a vertical tower observes a car moving
at a uniform speed towards the tower on a horizontal road.
, is : (2018/Online Set–2) If it takes 18 minute for the angle of depression of the car
2
to change from 300 to 450; then after this, the time taken (in
(a) 1 (b) 2 minute) by the car to reach the foot of the tower, is :
(c) 3 (d) 4 (2018/Online Set–3)
(a) 9 1 + 3
(b) 18 1 + 3
9
(c) 18 3 -1 (d)
2
3 -1
102 TRIGONOMETRY
(c) 11/32 (d) 11/16 8. 2 sin2 + 4 cos () sin sin + cos 2 () =
1 2 4 1
14. sin sin 3 sin 3 is equal to 20. If the equation sin = – 1/2 and tan = , then most
sin 3 3 3 3
common general values of is
4 3
(a) (b)
3 4 7 7
(a) 2n ± (b) 2n –
6 6
3
(c) (d) none of these
4 7
(c) 2n + (d) None of these
15. The number of distinct solution of 6
26. The number of solution in [0, /2] of the equation 33. cos 15 x = sin 5x if
cos 3x tan 5x = sin 7x is
(a) 5 (b) 7 (a) x n (b) x n
20 5 40 10
(c) 6 (d) none of these
3 3
27. sin x – cos2 x – 1 assumes the least value for the set of (c) x n (d) x n
20 5 40 10
values of x given by :
34. sin2x + 2 sin x cos x – 3cos2x = 0 if
(a) x = n + (–1)n + 1 (/6)
(a) tan x = 3 (b) tan x = –1
(b) x = n + (–1)n (/6)
(c) x = n + / 4 (d) x = n + tan–1 (–3)
(c) x = n + (–1)n (/3)
35. 5 sin2 x + 2
3 sin x cos x + 6cos x = 5 if
(d) x = n – (–1)n (/6) where n Z
28. If 2 cos2( + x) + 3 sin ( + x) vanishes then values of x lying (a) tan x = –1/ 3 (b) sin x = 0
in the interval from 0 to 2 are (c) x = n + / 2 (d) x = n + / 6
(a) x = /6 or 5 /6 (b) x = /3 or 5 /3 36. sin x + sin 2x + sin 3x = cos x + cos 2x + cos 3x if
(c) x = /4 or 5 /4 (d) x = /2 or 5 /2 (a) cos x = – 1/2 (b) sin 2x = cos 2x
29. 4 sin4x + cos4x = 1 if (c) x = n/ 2 + / 8 (d) x = 2n ± 2 / 3 (n I)
41. The value of cos y cos (/2 –x) – cos (/2 –y)
(a) n (b) 2n cos x + sin y cos (/2 –x) + cos x sin (/2 –y) is zero if
6 6
(a) x = 0 (b) y = 0
(c) n (1) n (d) n (c) x = y + /4 (d) y = x – 3/4
6 6
TRIGONOMETRY 105
3
(b) , ,
4 4 4 50. The number of values of in the interval ,
2 2
sec 2
(c) ,
satisfying the equation 3 = tan4 + 2 tan2 is
4 4
(a) 1 (b) 2
(d) none of these (c) 3 (d) none of these
51. If k1 = tan 27– tan and
45. If 4 sin2x – 8 sin x + 3 0, 0 x 2, then the solution set for
x is sin sin 3 sin 9
k2 , then
cos3 cos9 cos 27
5 (a) k1 = 2k2 (b) k1 = k2 + 4
(a) 0, (b) 0,
6 6
(c) k1 = k2 (d) none of these
5 7
5 5 52. If a = sin sin sin , and x is the solution of the
(c) , 2 (d) , 18 18 18
6 6 6
equation y = 2 [x] + 2 and y = 3 [x – 2], where [x] denotes the
integral part of x, then a is equal to
46. cos 4x cos 8x – cos 5x cos 9x = 0 if
(a) cos 12x = cos 14x (b) sin 13x = 0 1
(a) [ x ] (b)
[x]
(c) sin x = 0 (d) cos x = 0
(c) 2 [ x ] (d) [ x ]2
106 TRIGONOMETRY
54. If cot (– ), 3 cot , cot (+ ) are in A.P., and is not an
(a) tan = 1/ 3 (b) tan = – 1/ 3
integral multiple of , then sin cosec is equal to :
2 (c) tan = 3 (d) tan = – 3
3
59. Let f n tan (1 + sec ) (1 + sec 2)
(a) 2 (b) 2
2
(1 + sec 4)............ (1 + sec 2n ), then
2
(c) (d) none of these
3 (a) f 2 1 (b) f3 1
16 32
55. Which of following functions have the maximum value
unity?
(c) f 4 1 (d) f5 1
(a) sin2x – cos2x 64 128
sin 3 x cos3 x
1 61. The value(s) of the expression are
(b) cos (a 2 b 2 ) 1 cos x 1 sin x
2 2
(a) 2 cos x (b) 2 cos x
4 a 2 b2 4 4
(c) tan 2 2
2 a b
(c) 2 sin x (d) 2 sin x
a 2 b2 2 4 4
(d) cos ( )
2
TRIGONOMETRY 107
62. The equation sin 4x = a tan x, a > 0 66. Assertion : The function f (x) = min {sin x, cos x} takes the
(a) has no solution other than nif a > 4 4 20 43
value twice when x varies from to .
(b) has solutions which are not of the type nif 0 < a < 4 5 3 6
Reason : The periods of sin x and cos x are equal to 2.
(c) has solutions if a = 2009
(a) A (b) B (c) C
(d) has no solutions if a = 2008
(d) D (e) E
1
63. The equation |cot x| = cot x + , (n Z) 67. Assertion : The inequation tan x < 3 is equivalent to
sin x
1
cot x > .
2 3
(a) has a general solution (3n + 1)
3
1 1
Reason : If a < b then .
2 a b
(b) has a general solution (3n – 1)
3 (a) A (b) B (c) C
(c) is not defined if x = n (d) D (e) E
68. Assertion : The equation cos x + cos 2009 x = – 2 has
(d) cannot have a solution if cot x is positive
infinite solution.
Assertion Reason Reason : 2009 is an odd integer.
(A) If ASSERTION is true, REASON is true, REASON is a (a) A (b) B (c) C
correct explanation for ASSERTION. (d) D (e) E
(B) If ASSERTION is true, REASON is true, REASON is Using the following passage, solve Q.69 to Q.71
not a correct explanation for ASSERTION.
Passage –1
(C) If ASSERTION is true, REASON is false Given cos 2 cos 2
m m+1 n
.............. cos 2
(D) If ASSERTION is false, REASON is true
sin 2 n 1 m
(E) If both ASSERTION and REASON are false. n m 1
, where 2 k, n, m, k I
2 sin 2 m
Solve the following :
64. Assertion : If 2 sin 1 sin 1 sin , then lies
2 2
69. sin 9 . sin
11 sin 13
3 14 14 14
between 2n+ and 2n+
4 4
1 1
(a) (b)
3 64 64
Reason : If runs from to , then sin >0
2 4 4 2 1 1
(c) (d)
(a) A (b) B (c) C 8 8
(d) D (e) E
70. cos 23 cos 24 cos25 ........ cos 210 =
10 10 10 10
65. Assertion : The numbers sin 18° and – sin 54° are the roots
of same quadratic equation with integer co-efficients.
1 1
Reason : If x = 18°, then 5x = 90°, if y = – 54°, then (a) (b)
128 256
5y = –270°
(a) A (b) B (c) C 1 5 1 3
(c) sin (d) sin
512 10 512 10
(d) D (e) E
108 TRIGONOMETRY
1 1 1 1 1
(c) (d) (A) Over , (P) 2n , n 0
1024 2048 4 3 3
2 1 1
(A) cos 52 sin 2 22 (P) 1 7
2 2 (ii) cos2 x sin x for some x, then k must be equal
k 1
3 4 3 3 to
(B) cos2 cos 2 (Q)
5 5 4 2 80. The value of (1 – cot 23°) (1 – cot 22°) must be equal to
3
(C) sin 24° + cos 6° (R)
4
2 2 15 3
(D) sin 50 + cos 130° (S)
4
TRIGONOMETRY 109
1 1 2
(c) tan tan tan tan tan tan
2 2 2 2 2 2 (c) (d)
8 2 2
(d) None of the above
3
3. Given A = sin2 + cos4 , then for all real values of : 7. The expression 3 sin 4 sin 4 3
(1980) 2
3
(a) 1 A 2 (b) A 1
4 2 sin 6 sin 6 5 is equal to (1986)
2
13 3 13 (a) 0 (b) 1
(c) A 1 (d) A
16 4 16
(c) 3 (d) sin 4 + cos 6
1 8. The number of all possible triplets (a1, a2, a3) such that
4. The equation 2 cos2 x sin2 x = x2 + x–2, x has : a1 + a2 cos (2x) + a3 sin2 (x) = 0 for all x is : (1987)
2 9
(1980) (a) zero (b) one
(a) no real solution (c) three (d) inifinite
(b) one real solution 9. The smallest positive root of the equation, tan x – x = 0 lies
in : (1987)
(c) more than one real solution
(d) none of these
(a) 0, (b) ,
2 2
3 3
(c) , (d) ,2
2 2
110 TRIGONOMETRY
The value of the expression 18. The number of values of x in the interval [0, 5] satisfying
10. 3 cosec 20º – sec 20º is
the equation 3 sin2 x –7 sin x +2 = 0 is : (1998)
equal to : (1988)
(a) 0 (b) 5
(a) 2 (b) 2 sin 20º/ sin 40º
(c) 6 (d) 10
(c) 4 (d) 4 sin 20º/ sin 40º
19. Which of the following numbers is rational ? (1998)
11. The general solution of sin x –3 sin 2x + sin 3x
= cos x –3 cos 2x + cos 3x is : (1989) (a) sin 15º (b) cos 15º
(c) sin 15º cos 15º (d) sin 15º cos 75º
n
(a) n + (b)
8 2 8
20. If and , then tan equals : (2001)
2
n 3
(c) (–1)n (d) 2n + cos–1 (a) 2 (tan + tan ) (b) tan + tan
2 8 2
(c) tan + 2 tan (d) 2 tan + tan
12. The equation (cos p –1) x2 + (cos p) x + sin p = 0 in the
variable x, has real roots. Then p can take any value in the 21. The number of integral values of k for which the equation
interval : (1990) 7 cos x + 5 sin x = 2k + 1 has a solution, is : (2002)
(a) (0, 2) (b) (–, 0) (a) 4 (b) 8
(c) 10 (d) 12
(c) – , (d) (0, ) 1 1
2 2 22. Given both and are acute angles sin , cos ,
2 3
13. In a triangle ABC, angle A is greater than angle B. If the
then the value of + belongs to (2004)
measures of angles A and B satisfy the equation
3 sin x –4 sin3 x –k = 0, 0 < k < 1, then the measure of angle 2
C is : (1990) (a) , (b) ,
3 6 2 3
(a) /3 (b) /2
(c) 2/3 (d) 5/6 2 5 5
(c) , (d) ,
14. The number of solutions of the equation 3 6 6
sin (e)x = 5x + 5–x is : (1991) 23. The number of ordered pairs (), where (–)
(a) 0 (b) 1 1
(c) 2 (d) infinitely many satisfying cos ( – =1 and cos is :
e
15. Number of solution of the equation (2005)
tan x + sec x = 2 cos x lying in the interval [0, 2] is:
(a) 0 (b) 1
(1993)
(c) 2 (d) 4
(a) 0 (b) 1
24. The set of value of satisfying the inequation
(c) 2 (d) 3
2 sin2 – 5 sin +2 > 0, where 0 < < 2 is: (2006)
16. The general value of satisfying the equation
2 sin2 – 3 sin –2 = 0, is : (1995) 5 5
n n (a) 0, ,2 (b) 0, , 2
(a) n+ (–1) /6 (b) n+ (–1) /2 6 6 6 6
(c) n+ (–1)2 5/6 (d) n+ (–1)n 7/6
2
17. 3 (sin x – cos x)4 + 6 (sin x + cos x)2 + 4 (sin 6 x + cos6 x) equal (c) 0, , 2 (d) none of these
to : (1995) 3 3
(a) 11 (b) 12
(c) 13 (d) 14
TRIGONOMETRY 111
13
1
25. Let 0, and t 1 = (tan ) cot , t 2 = (tan ) cot , 31. The value of k 1 k
is equal to
4 k 1
sin sin
6 6
t3 = (cot )tan and t4 = (cot )cot , then (2006)
(a) t1 > t2 > t3 > t4 (b) t4 > t3 > t1 > t2 (2016)
(c) t3 > t1 > t2 > t4 (d) t2 > t3 > t1 > t4 (a) 3 3 (b) 2 3 3
26. The number of solutions of the pair of equations
2sin2 – cos 2 =0 & 2cos2 – 3sin =0 in the interval (c) 2 3 1
(d) 2 2 3
[0, 2] is (2007) Multiple Answer Question
(a) zero (b) one 32. The values of lying between = 0 and = /2 and
(c) two (d) four satisfying the equation
27. Let P = {: sin – cos = 2 cos } and 1 sin 2 cos 2 4sin 4
2 2
Q = {: sin + cos = 2 sin } be two sets. Then sin 1 cos 4sin 4 0, is (1988)
(2011) sin 2 2
cos 1 4 sin 4
(a) P Q and Q – P (b) Q P (a) 7/24 (b) 5/24
(c) P Q (d) P = Q (c) 11/24 (d) /24
28. In a triangle the sum of two sides is x and the product of the
same two sides is y. If x2 – c2 = y, where c is the third side of
33. For 0 < < /2, if x cos 2n , y sin 2n ,
the triangle, then the ratio of the in-radius to the circum- n 0 n 0
radius of the triangle is (2014)
5
(c) (d) (c) f 4 1 (d) f5 1
64 128
112 TRIGONOMETRY
the 35
37. For 0 , solution(s) of (b) the radius of circumcircle of the triangle XYZ is 6
2 6
6
(m 1) m X Y Z 4
(c) sin sin sin
cos ec
m 1 4
cos ec
4 2 is/are
4
2 2 2 35
2 X Y 3
(2009) (d) sin
2 5
41. Let and be non zero real numbers such that 2(cos –
(a) (b)
4 6 cos ) + cos cos = 1. Then which of the following is/
are true? (2017)
5
(c) (d)
12 12
(a) tan 3 tan 0
38. Let , [0, 2] be such that 2 cos (1 – sin ) = sin2 2 2
tan cot cos 1, tan (2 – ) > 0 and (b) 3 tan tan 0
2 2 2
2
3
1 sin . Then, cannot satisfy (2012)
2 (c) tan 3 tan 0
2 2
4
(a) 0 (b)
2 2 3 (d) 3 tan tan 0
2 2
4 3 3 Match the Column
(c) (d) 2
3 2 2 42. (sin 3)/(cos 2) is (1992)
1 Column I Column II
39. In a PQR, P is the largest angle and cos P . Further in
3 (A) positive (p) (13/48, 14/48)
circle of the triangle touches the sides PQ, QR and RP at N, (B) negative (q) (14/48, 18/48)
L and M respectively, such that the lengths of PN, QL and (r) (18/48, 23/48)
RM are consecutive even integers. Then, possible length(s)
of the side(s) of the triangle is (are) (s) (0, /2)
(2013)
(a) 16 (b) 18
(c) 24 (d) 22
TRIGONOMETRY 113
Integer Answer Type Question 53. General value of satisfying the equation
43. The number of all possible values of , where 0 < < , for tan2 + sec 2=1 is.... . (1996)
which the system of equations True/False
(y + z) cos 3= (xyz) sin 3 54. If tan A = ( 1 – cos B) / sin B, then tan 2 A = tan B.
(1983)
2cos 3 2sin 3
x sin 3 55. There exists a value of between 0 and 2that satisfies the
y z
equation sin4– 2 sin2 – 1 = 0. (1984)
and (xyz) sin 3 = (y + 2z) cos 3y sin 3have a solution Subjective Question
(x0, y0, z0) with y0z0 0, is ..... (2010)
56. Solve 2 (cos x + cos 2x) + (1 + 2 cos x) sin 2x
44. The number of distinct solution of the equation
= 2 sin x, – x (1978)
5
cos2 2x + cos4 x + sin4 x + cos6 x + sin6 x = 2 in the interval
4 57. Prove that 5cos 3cos + 3 lies between –4 and 10.
3
[0, 2] is (2015)
Analytical and Descriptive Questions (1979)
45. Prove that sin x . sin y. sin (x–y) + sin y. sinz.sin (y–z) 58. Given + + = , prove that sin + sin – sin2
2 2
67. Prove that tan + 2 tan 2 + 4 tan 4 + 8 cot 8 = cot . 71. Find the smallest positive number p for which the equation
(1988) cos (p sin x) = sin (p cos x) has a solution x [0, 2].
68. ABC is a triangle such that (1995)
72. Find number of values of in the interval
1
sin (2A + B) = sin (C – A) = –sin (B + 2C) .
2 , satisfying the equation
2 2
If A, B and C are in Arithmetic Progression, determine the 2
values of A, B and C. (1990) (1–tan ) (1 + tan ) sec2 2tan 0.
69. If exp {(sin2x + sin4 x + sin6 x +.......) loge 2}, satisfies the (1996)
equation x 2 – 9x + 8 = 0, find the value of sin x cos3x
cos x 73. Prove that the values of the function do not lie
,0x . (1991) sin 3x cos x
cos x sin x 2
1
70. Determine the smallest positive value of x (in degrees) for between and 3 for any real x. (1997)
3
which tan (x + 100°) = tan (x + 50°) tan (x) tan (x – 50°)
74. In any triangle prove that
(1993)
A B C A B C
cot cot cot cot cot cot (2000)
2 2 2 2 2 2
115 TRIGONOMETRY
ANSWER KEY
EXERCISE - 1 : BASIC OBJECTIVE QUESTIONS
1. (b) 2. (d) 3. (d) 4. (c) 5. (a) 6. (a) 7. (b) 8. (c)
9. (b) 10. (c) 11. (d) 12. (d) 13. (a) 14. (b) 15. (a) 16. (b)
17. (d) 18. (a) 19. (a,d) 20. (d) 21. (b) 22. (b) 23. (b) 24. (a)
25. (a) 26. (b) 27. (c) 28. (a,b,c,d) 29. (c) 30. (b,c) 31. (b) 32. (a)
33. (a) 34. (a) 35. (d) 36. (b) 37. (a) 38.(b) 39. (a) 40. (b)
41 (b) 42 (a) 43. (b) 44. (b) 45. (b) 46. (c) 47. (b) 48. (a)
49 (b) 50 (b,c) 51. (b,d) 52. (b) 53. (d) 54. (b) 55. (c) 56. (d)
57. (c) 58. (b) 59. (d) 60. (a) 61. (d) 62. (b) 63. (b) 64. (d)
65. (c) 66. (a) 67. (c) 68. (b) 69. (a) 70. (c)
EXERCISE - 2 : PREVIOUS YEAR JEE MAINS QUESTIONS
1. (a) 2. (b) 3. (c) 4. (c) 5. (b) 6. (a) 7. (d) 8. (a)
9. (b) 10. (c) 11. (b) 12. (b) 13. (a) 14. (c) 15. (b) 16. (c)
17. (a) 18. (c) 19. (c) 20. (a) 21. (b) 22. (c) 23. (c) 24. (c)
25. (c) 26. (b) 27. (c) 28. (d) 29. (a) 30. (b) 31. (a) 32. (b)
33. (a) 34. (a) 35. (c) 36. (b) 37. (c) 38. (c) 39. (b) 40. (b)
41. (d) 42. (c) 43. (d) 44. (c) 45. (c) 46. (c) 47. (b) 48. (a)
49. (c) 50. (c) 51. (a)
EXERCISE - 3 : ADVANCED OBJECTIVE QUESTIONS
1. (c) 2. (a) 3. (b) 4. (c) 5. (a) 6. (b) 7. (c) 8. (c)
9. (b) 10. (b) 11. (d) 12. (b) 13. (b) 14. (c) 15. (d) 16. (b)
17. (d) 18. (c) 19. (b) 20. (c) 21. (a) 22. (b) 23. (a) 24. (b)
25. (b) 26. (c) 27. (a,d) 28. (a) 29. (a,b) 30. (c) 31. (a) 32. (c)
33. (a,b,c,d) 34. (c,d) 35. (a,c) 36. (a,b,c,d) 37. (a,b) 38. (d) 39. (c) 40. (b)
41. (d) 42. (d) 43. (c) 44.(a) 45. (d) 46. (a,b,c) 47. (b) 48. (a,b)
49. (d) 50. (d) 51. (a) 52. (b) 53. (d) 54. (b) 55. (a,b,c,d) 56. (a,c,d)
57. (b) 58. (a,b,c,d) 59. (a,b,c,d) 60. (a,b,c) 61. (a,d) 62. (a,b,c) 63. (a,c,d) 64. (b)
65. (a) 66. (d) 67. (e) 68. (a) 69. (c) 70. (b) 71. (c) 72. (b)
73. (c) 74. (d) 75. A Q; B R; C S; D P 76. A Q; B R; C P
77. 0018 78. 0030 79. 0018 80. 0002
EXERCISE - 4 : PREVIOUS YEAR JEE ADVANCED QUESTIONS
1. (b) 2. (a) 3. (b) 4. (a) 5. (c) 6. (c) 7. (b) 8. (d)
9. (c) 10. (c) 11. (b) 12. (d) 13. (c) 14. (a) 15. (c) 16. (d)
17. (c) 18. (c) 19. (c) 20. (c) 21. (b) 22. (b) 23. (d) 24. (a)
25. (b) 26. (c) 27. (d) 28. (b) 29. (d) 30. (c) 31. (c) 32. (a,c)
33. (b,c) 34. (a,b) 35. (a,b,c,d) 36. (a,b) 37. (c,d) 38. (a,c,d) 39. (b,d) 40. (a,c,d)
56
41. (a,c) 42. A–r; B–p 43. (0003) 44. (0008) 46. 47. 6 48. cos (log )
33
5 1 1
49. no solution 50. x 0, , 51. 52. 53. m, n
6 2 6 64 8 3
3 3
54. True 55. False 56. x , , , , 60. ,cos ,cos ,cos
2 3 3 8 8 4 4 8 8
3 2
66. n or n (1) n
n n
64. {x : x n} x : x n (1) 10 x : x n (1) 10 65. ,
3 3 6
3 1
68. A = 45 º, B = 60º, C = 75º 69. 70. 30° 71. 72. 4
2 2 2
Dream on !!
04
STATISTICS
117 STATISTICS
STATISTICS
1. INTRODUCTION : n
fi x i
f x f x .... f n x n
An average or a central value of a statistical series in the x 1 1 2 2 i 1
n
f1 f 2 ...f n
value of the variable which describes the characteristics of fi
i 1
the entire distribution.
The following are the five measures of central tendency.
f x A
(1) Arithmetic Mean (ii) Short cut method : Arithmetic mean x A
f
(2) Geometric Mean
Where A = assumed mean, f = frequency and x - A = deviation
(3) Harmonic Mean of each item from the assumed mean.
(4) Median (3) Properties of arithmetic mean
(5) Mode (i) Algebraic sum of the deviations of a set of values from
their arthmetic mean is zero. If xi/fi, i = 1, 2, ..., n is the
2. ARITHMETIC MEAN :
frequency distribution, then
Arithmetic mean is the most important among the n
mathematical mean. fi x i x 0, x being the mean of the distribution.
i 1
According to Horace Secrist,
(ii) The sum of the squares of the deviations of a set of
“The arithmetic mean is the amount secured by dividing the
values is minimum when taken about mean.
sum of values of the items in series by their number”.
(1) Simple arithmetic mean in individual series (Ungrouped (ii) Mean of the composite series : If x i (i = 1,2,...,k) are the
data) means of k-component series of sizes ni, (i = 1, 2, ..., k)
(i) Direct method : If the series in this case be x1,x2,x3,...., xn respectively, then the mean x of the composite series
then the arithmetic mean x is given by obtained on combining the component series is given by
d
Arithmetic mean x A
n
, Where, A = assumed mean,
3. GEOMETRIC MEAN:
d = deviation from assumed mean = x - A, where x is the
individual item, d = sum of deviations and n= number of If x1, x2, x3, ...xn are n values of a variate x, none of them being
items. zero, then geometic mean (G.M.) is given by
G.M. = (x 1 , x 2 , x 3 , ....x n) 1/n log (G.M.)
(2) Simple arithmetic mean in continous series (Grouped
data) 1
log x1 log x 2 ... log x n .
(i) Direct method : If the terms of the given series be x1, x2, 2
....xn and the corresponding frequencies be f1, f2, f3...fn, then In case of frequency distribution G.M. of n values x1, x2, ..., xn
the arithmetic mean x is given by,, of a variate x occuring with frequency f1, f2,..., fn is given by
1
G.M. x1f1 .x f22 ,....,fnn N , where N = f1 + f2 + ... + fn.
118 STATISTICS
f1 f 2 f3 ..... f n N
ten H.M. C
f1 f 2 f 2 i
....... n Median = u , where u = upper limit of the
f
x
1 x 2 x n
median class.
Note : A.M. gives more weightage to larger values whereas
n
G.M. given more weightage to smalle. M fi
i 1
5. MEDIAN :
As median divides a distribution into two equal parts,
Median is defined as the value of an item or observation similarly the quartiles quantiles, deciles and percentiles
above or below which lies on an equal number of observation divide the distribution respectively into 4, 5, 10 and 100
i.e., the median is the central value of the set of observations equal part. The j th quartile is given by
provided all the observations are arranged in the ascending
or descending orders. N
j 4 C
(1) Calculation of median Qj l i; j 1, 2,3.Q1 is the lower quartile, Q2 is
f
(i) Individual series : If the data is raw, arrange in ascending
or descending order. Let n be the number of observations.
the median and Q3 is called the upper quartile.
th (2) Lower quartile
n 1
If n is odd, Median = value of item.
2 th
(i) Discrete series : Q1 size of n 1 item
If n is even, median 4
th th
1 n n
value of item + value of 1 item N
2 2 2 C
4
(ii) Continous series : Q1 i
f
(ii) Discrete series : In this case, we first find the cumulative
frequencies of the variables arragned in ascending or (3) Upper quartile
descending order and the median is given by
th
3 n 1
n 1
th (i) Discrete series : Q3 size of item
Median = observation, where n is the cumulative 4
2
frequency. 3N
C
(iii) For grouped or continous distributions : In this case, (ii) Continous series : Q3 4 i
following formula can be used f
Nk
C
Pk 100 i
f
(i) Mean deviation from ungerouped data (or indiviadual N = f = The total frequency
series)
STATISTICS 121
Short cut method Variance of the combined series : If n1; n2 are the sizes, x1 ; x 2
2
the means and 1 ; 2 the standard deviation of two series,
2
fd 2 fd d 2 d
(i) (ii)
N N N N 1
2
then
n1 12 d12 n 2 22 d 22
n1 n 2
where, d = x – A = Deviation from the assumed mean A
f = Frequency of the item n1x1 n 2 x 2
Where, d1 x1 x, d 2 x 2 x .
N = f = Sum of frequencies n1 n 2
(4) Square deviation
(i) Root mean square deviation
1 n 2
S fi (x A)
4 i 1 Range is widely used in satistical series relating to quality
control in production.
where A is any arbitrary number and S is called mean square
Standard deviation Range i.e., variance (Range)2.
deviation.
Empirical relation between measures of dispersion.
(ii) Relation between S.D. and root mean square deviation :
If be the standard deivation and S be the root mean square 4
deviation. Mean deviation = (standard deviation)
5
Then S2 = 2 + d2.
n2 1
S.D. 100 100 . S.D. of first n natural number is .
x 12
SOLVED EXAMPLES
Example 1 : Example 3 :
If the mean of the distribution is 2.6, then the value of y is If the mean of the set of number x1,x2, x3, ...,xn is x , then the
mean of the number xi + 2i, 1 i n is
Variate x :1 2 3 4 5
Frequency f of x: 4 5 y 1 2 (a) x 2n (b) x n 1
(a) 24 (b) 13 (c) 8 (d) 3 (c) x 2 (d) x n
n
n xi n
fi x i Solution (b) : We know that x i 1
i.e., x i nx
i 1
Solution (c): We know that, Mean n
n i 1
fi
i 1
n n n
x i 2i xi 2 i nx 2 1 2 ...n
i 1
i 1 i 1
n n n
1 4 2 5 3xy 4 1 5 2
i.e. 2.6 or 31.2 + 2.6y = 28
4 5 y 1 2
n n 1
nx 2
+ 3y or 0.4y = 3.2 y = 8 2 x n 1
n
Example 2 :
Example 4 :
In a class of 100 students there are 70 boys whose average
The harmonic mean of 4, 8, 16 is
marks in a subject are 75. If the average marks of the
(a) 6.4 (b) 6.7 (c) 6.85 (d)7.8
complete class are 72, then what are the average marks of
the girls.
3 48
(a) 73 (b) 65 (c) 68 (d) 74 Solution (c): H.M. of 4, 8, 16 6.85
1 1 1 7
4 8 16
Solution (b) : Let the average marks of the girls students be x,
then Example 5 :
x1 x 2 x 3 ...x n
Solution (b): M i.e.,
n
124 STATISTICS
Example 8 :
nM x1 x 2 x 3 ...x n 1 x n
nM x n x1 x 2 x 3 ...x n 1 The median of a set of 9 distinct observation is 20.5. If
nM x n x' x1 x 2 x 3 x n 1 x ' each of the largest 4 observation of the set is increased by
2. then the median of the new set.
n n
(a) Is increased by 2
(b) is decreased by 2
nM x n x '
New average (c) Is two times the original median
n
(d) Remains the same as that of the original set
Example 6 :
Mean of 100 items is 49. It was discovered that three items 9 1 th
which should have been 60, 70, 80 were wrongly read as Solution (d): n = 9, then median term 5 term. Since
2
40, 20, 50 respectively. The correct mean is
last four observation are increased by 2.
1 The median is 5th observation which is remaining
(a) 48 (b) 82 (c) 50 (d) 80
2 unchanged.
There will be no change in median.
Solution (c):
Example 9 :
Sum of 100 items = 49 × 100 = 4900
Sum of items added = 60 + 70 + 80 = 210 Compute the median from the following table
Sum of items replace = 40 + 20 + 50 = 110
New sum = 4900 + 210 - 110 = 5000 Marks obtained No. of students
0-10 2
5000 10-20 18
Correct mean= 50
100 20-30 30
30-40 45
Example 7 : 40-50 35
50-60 20
The following date gives the distribution of height of 60-70 6
students 70-80 3
Height (in cm) 160 150 152 161 156 154 155
Number of students 12 8 4 4 3 3 7
(a) 36.55 (b) 35.55
The median of the distribution is (c) 40.05 (d) None of these
(a) 154 (b) 155 (c) 160 (d) 161
Solution (a):
Solution (b): Arranging the data in ascending order of magnitude,
Marks obtained No. of Cumulative
we obtain students Frequency
0-10 2 2
H eig h t (in c m ) 150 152 154 155 156 160
N u m b er o f stu d e nts 8 4 3 7 3 12
10-20 18 20
C u m u lativ e 8 12 15 22 25 37 20-30 30 50
freq u en c y 30-40 45 95
40-50 35 130
Here, total number of items is 41, i.e., an odd number. Hence, 50-60 20 150
41 1 60-70 6 156
the median is th i.e. 21st item 70-80 3 159
2
For cumulative frequency table, we find that median i.e., n = f = 159
21st item is 155. Hence n = 159, which is odd.
(All items from 16 to 22nd are equal each = 155)
STATISTICS 125
1 1 Example 12 :
Median number n 1 159 1 80, which is in In a series of 2n observations, half of them equal a and
2 2
the class 30-40 (see the row of cumulative frequency 95, remaining half equal -a. If the standard deviation of the
which contains 80). observations is 2, then |a| equals
Example 10 : Example 13 :
A batsman scores runs in 10 inning 38, 70, 48, 34, 42, 55, If is the mean of distribution (yi, fi), then fi(yi - ) =
63, 46, 54, 44, then the mean deviation is
(a) M.D. (b) S.D. (c) 0 (d) Relative frequency
(a) 8.6 (b) 6.4 (c) 10.6 (d) 9.6
Solution (c): We have fi(yi - ) = fiyi – fi = fi – fi = 0
Solution (a): Arranging the given data in ascending order, we
have fi yi
34, 38, 42, 44, 46, 48, 54, 55, 63, 70 fi
46 48 Example 14 :
Here median M 47
2
What is the standard deviation of the following series
( n = 10, median is the mean of 5th and 6th items)
Measurements 0-10 10-20 20-30 30-40
Mean deviation
Frequency 1 3 4 2
xi M x i 47 13 9 5 3 1 1 7 8 16 23 (a) 81 (b) 7.6 (c) 9 (d) 2.26
8.6
n 10 10
Solution (c):
Example 11 :
S.D. of data is 6 when each observation is increased by 1, Class Frequency y1 yi A f1u1 f1u12
i , A 25
then the S.D. of new data is 10
(a) 5 (b) 7 (c) 6 (d) 8 0–10 1 5 –2 –2 4
10–20 3 15 –1 –3 3
Solution (c): S.D. and variance of data is not changed, when each 20–30 4 25 0 0 0
observation is increased (OR decreased) by the same 30–40 2 35 1 2 2
constant. 10 –3 9
f u 2 f u 2 2
2 2
c i i i i
f i f
i
126 STATISTICS
Example 16 :
2
9 3 2
10 90 9 81 9
10 10 ax b
The S.D. of a variate x is . The S.D. of the variate
c
Example 15 :
where a, b, c are constant, is
In an experiment with 15 observation on x, the following
a a
resultswereavailablex2 = 2830, x = 170. On observation (a) (b)
that was 20 was found to be wrong and was replaced by c c
the correct value 30. Then the corrected variance is
a2
(a) 78.00 (b) 188.66 (c) 177.33 (d) 8.33 (c) 2 (d) None of these
c
Solution (a): x = 170, x2 = 2830
Increase in x = 10 then x1 = 170 + 10 = 180 Solution (b):
Increase in x2 = 900 - 400 = 500, then x’ = 2830 + 500 = 3330 ax b a b
Let y i.e., y x i.e. y = Ax + B, where
2 2 c c c
1 x ' 3330 180
Variance x '2 222 144 78
n n 15 15 a b
A ,B
c c
y Ax B
2 2
yy A xx yy
A2 x x
2 2
y y A 2 x x n.2y A.n2y 2y A 2 2x
a
y A x y x
c
a
Thus, new S.D. .
c
127 STATISTICS
15. Statement I The average marks of boys in a class is 52 21. The standard deviation of 25 numbers is 40. If each of the
and that of girls is 42. The average marks of boys and numbers is increased by 5, then the new standard
girls combined is 50. The percentage of boys in the class deviation will be
is 80%. (a) 40 (b) 45
Statement II Mean marks scored by the students of a
class is 53. The mean marks of the girls is 55 and the mean 21
(c) 40 (d) None of these
marks of the boys is 50. The percentage of girls in the 25
class is 64%. 22. The median of a set of 9 distinct observations is 20.5. If
(a) Only statement I is true (b) Only statement II is true each of the largest 4 observations of the set is increased
(c) Both statements are true(d) Both statements are false by 2, then the median of the new set is
16. The variance of the first n natural numbers is (a) increased by 2 (b) decreased by 2
(c) two times the original median(d) remains the same as
n2 1 n2 1 that of original set
(a) (b)
12 6 23. The mean age of a combined group of men and women is
25 yrs. If the mean age of the group of men is 26 and that
n2 1 n2 1 of the group of women is 21, then the ratio of men and
(c) (d)
6 12 women in the group is
17. If v is the variance and is the standard deviation, then (a) 1 : 4 (b) 4 : 1
(c) 3 : 1 (d) 1 : 3
(a) v2 (b) v 2
24. The mean of five observations is 4 and their variance is
1 1 5.2. If three observations are 1, 2, and 6, the other two are
(c) v (d) v
2 (a) 2 and 9 (b) 3 and 8
(c) 4 and 7 (d) 5 and 6
18. If each observation of a raw data whose variance is 2 ,
25. Consider any set of observations x1, x2, x3,....,x101; it being
is increased by , then the variance of the new set is
given that x1 < x2 < x3 < ... <x101; then the mean deviation of
(a) 2 (b) 2 2 this set of observations about a point k is minimum when
k equals
(c) 2 (d) 2 2 (a) x1 (b) x51
(a) 2 (b) 2 2 26. Statement I The mean and variance for first n natural
(c) 2 (d) 2 2 n 1 n2 1
numbers are and , respectively..
2 12
20. If the variance of observations x1, x2, ....... xn is 2 , then
Statement I The mean and variance for first 10 positive
the variance of observations ax1, ax2, ......, axn, a 0 is multiples of 3 are 16.5 and 74.25, respectively.
(a) 2 (b) a2 (a) Only statement I is true (b) Only statements II is true
(c) Both statements are true(d) Both statements are false
2 27. The standard deviation of the data 6, 5, 9, 13, 12, 8, 10 is
(c) a 2 2 (d)
a2
52 52
(a) (b)
7 7
(c) 6 (d) 6
STATISTICS 129
28. Variance of the data 2, 4, 5, 6, 8, 17 is 23.33. Then, variance Further, another set of 15 observations x1, x2......,xn (also
of the data 4, 8, 10, 12, 16, 34 will be n 15
(a) 23.33 (b) 25.33 in seconds) is now available and we have x i 279
i 1
(c) 93.32 (d) 98.32
15
29. The mean of 100 observations is 50 and their standard 2
and x i 5524. The standard deviation of all 40
deviation is 5. The sum of squares of all observations is i 1
1 2
(c) (d)
n n
STATISTICS 131
12. Statement-1 : The variance of first n even natural numbers 16. All the students of a class performed poorly in
Mathematics. The teacher decided to give grace marks of
n2 1 10 to each of the students. Which of the following statistical
is
4 measures will not change even after the grace marks were
Statement-2 : The sum of first n natural numbers is given ? (2013)
(a) median (b) mode
n (n 1)
and the sum of squares of first n natural numbers (c) variance (d) mean
2
17. The mean of a data set consisting of 20 observations is 40.
n (n 1) (2 n 1)
is (2009) If one observation 53 was wrongly recorded as 33, then
6 the correct mean will be (2013/Online Set 1)
(a) Statement 1 is true, Statement 2 is true; Statement 2 is (a) 41 (b) 49
not a correct explanation for Statement 1.
(c) 40.5 (d) 42.5
(b) Statement 1 is true, Statement 2 is false
18. Mean of 5 observations is 7. If four of these observations
(c) Statement 1 is false, Statement 2 is true. are 6, 7, 8, 10 and one is missing, then the variance of all
(d) Statement 1 is true, Statement 2 is true; Statement 2 is the five observations is (2013/Online Set 2)
a correct explanation for Statement 1 (a) 4 (b) 6
13. For two data sets, each of size 5, the variances are given to (c) 8 (d) 2
be 4 and 5 and the corresponding means are given to be 2
19. If the median and the range of four numbers
and 4, respectively. The variance of the combined data set
[x, y, 2x + y, x – y], where 0 < y < x < 2y, are 10 and 28
is (2010)
respectively, then the mean of the numbers is
5 11 (2013/Online Set 3)
(a) (b)
2 2 (a) 18 (b) 10
(c) 5 (d) 14
13
(c) 6 (d) 20. In a set of 2n observation, half of them are equal to a and
2
the remaining half are equal to –a. If the standard deviation
14. If the mean deviation about the median of the numbers a, of all the observations is 2, then the value of |a| is
2a, ..........., 50a is 50, then |a| equals (2011) (2013/Online Set 4)
(a) 4 (b) 5
(a) 2 (b) 2
(c) 2 (d) 3
(c) 4 (d) 2 2
15. Let x1 , x 2 , ......., x n be n observations, and let x be their
21. The variance of first 50 even natural numbers is (2014)
arithmetic mean and 2 be their variance. (2012)
(a) 833 (b) 437
2
Statement 1 : Variance of 2x1 , 2x 2 , ........., 2 x n is 4 . 437 833
(c) (d)
Statement 2 : Arithmetic mean of 2x1 , 2x 2 , ......, 2x n is 4 4
22. In a set of 2n distinct observations, each of the observation
4x.
below the median of all the observations is increased by 5
(a) Statement 1 is true, Statement 2 is true; Statement 2 is and each of the remaining observations is decreased by 3.
not a correct explanation for Statement 1. Then the mean of the new set of observations:
(b) Statement 1 is true, Statement 2 is false (2014/Online Set 1)
(c) Statement 1 is false, Statement 2 is true. (a) increases by 1 (b) decreases by 1
(d) Statement 1 is true, Statement 2 is true; Statement 2 is (c) decreases by 2 (d) increases by 2
a correct explanation for Statement 1
132 STATISTICS
23. Let X and M.D. be the mean and the mean deviation 29. The mean age of 25 teachers in a school is 40 years. A
teacher retires at the age of 60 years and a new teacher is
about X of n observations x i , i 1, 2,......., n. If each of appointed in his place. If now the mean age of the teachers
the observations is increased by 5, then the new mean in this school is 39 years, then the age (in years) of the
and the mean deviation about the new mean, respectively, newly appointed teacher is (2017/Online Set 1)
are : (2014/Online Set 3) (a) 25 (b) 35
(a) X, M.D. (b) X 5, M.D. (c) 30 (d) 40
30. The sum of 100 observations and the sum of their squares
(c) X, M.D. 5 (d) X 5, M.D. 5 are 400 and 2474, respectively. Later on, three observations
3, 4 and 5, were found to be incorrect. If the incorrect
24. Let x, M and 2 be respectively the mean, mode and observations are omitted, then the variance of the
variance of n observations x1 , x 2 ..........., x n and remaining observations is (2017/Online Set 2)
(a) 8.00 (b) 8.25
di x i a, i 1, 2, .........., n, where a is any number..
(c) 9.00 (d) 8.50
(2014/Online Set 4)
9 9
2
Statement-1 : Variance of d1 , d 2 , .............., d n is 2 31. If (x i 5) 9 and (x i 5) 45, then the
i 1 i 1
Statement-2 : Mean and mode of d1 , d 2 , .........., d n are
standard deviation of the 9 items x1 , x 2 , ........, x 9 is :
x a and – M – a, respectively.. (2018)
(a) Statement-1 and Statement-2 are both false (a) 9 (b) 4
(b) Statement-1 and Statement-2 are both true (c) 2 (d) 3
(c) Statement-1 is true and Statement-2 is false 32. The mean of a set of 30 observations is 75. If each
(d) Statement-1 is false and Statement-2 is true observation is multiplied by a non-zero number and then
each of them is decreased by 25, their mean remains the
25. The mean of the data set comprising of 16 observations is same. Then is equal to : (2018/Online Set 1)
16. If one of the observation valued 16 is deleted and three
new observations valued 3, 4 and 5 are added to the data, 1 2
then the mean of the resultant data is : (a) (b)
3 3
(2015)
(a) 16.0 (b) 15.8 4 10
(c) (d)
3 3
(c) 14.0 (d) 16.8
26. If the standard deviation of the numbers 2, 3, a and 11 is 33. If the mean of the data : 7, 8, 9, 7, 8, 7, , 8 is 8, then the
3.5, then which of the following is true ? (2016) variance of this data is : (2018/Online Set 2)
ANSWER KEY
EXERCISE - 1 : BASIC OBJECTIVE QUESTIONS
1. (c) 2. (d) 3. (b) 4. (b) 5. (d) 6. (a) 7. (d) 8. (c) 9. (b) 10. (a)
11. (c) 12. (b) 13. (c) 14. (a) 15. (a) 16. (a) 17. (b) 18. (a) 19. (b) 20. (c)
21. (a) 22. (d) 23. (b) 24. (c) 25. (b) 26. (b) 27. (a) 28. (c) 29. (c) 30. (a)
31. (c) 32. (a) 33. (c) 34. (c) 35. (c) 36. (b) 37. (d) 38. (a) 39. (c) 40. (a)
Dream on !!
05
MATHEMATICAL REASONING
135 MATHEMATICAL REASONING
MATHEMATICAL REASONING
Truth Table
10. CONTRAPOSITIVE, CONTRADICTION AND
CONVERSE OF A CONDITIONAL STATEMENT:
8. TYPES OF “OR”:
If p and q are two statements then
(i) Exclusive OR : If in statement p v q i.e., p or q, happening
of any one of p, q excludes the happening of the other Let p q then
then it is exclusiver or. Here both p and q cannot occur (i) (Contrapositive of p q) is (~q ~p)
together. For example in statement “I will go to delhi either (ii) Contradiction of p q) is (q p)
by bus or by train”, the use of ‘or’ is exclusive.
(iii) (Converse of p q) is (q p)
(ii) Inclusive OR : If in statement p or q, both p and q can
also occur together then it is inclusive or. The statement
‘In senior secondary exam you can take optional subject
as physical education or computers’ is an example of use
A statement and its contrapositive convey the same
of inclusive OR.
meaning.
IMPLICATION :
11. BICONDITIONAL CONNECTIVE “IF AND ONLY IF”:
There are three types of implications which are “if... then”,
“Only if” and “if and only if” If p and q are any two statements then the compound
statement in the form of “p if and only if q” is called a
9. CONDITIONAL CONNECTIVE ‘IF... THEN’: biconditional statement and is written in symbolic form as
p q or p q.
If p and q are any two statements then the compound
statement in the form “if p then q” is called a conditional Statement p q reveals the following facts
statement. The statement “If p then q” is denoted by p (i) p if and only if q
q or p q (to be read as p implies q).In the implication p
(ii) q if and only if p
q, p is called the antecedent (or the hypothesis) and q
the consequent (or the conclusion) (iii) p is necessary and sufficient condition for q
If p then q reveals the following facts : (iv) q is necessary and sufficient condition for p
we observe that last column is always false. Hence p ^ ~p (iv) Negation of biconditional : ~(p q) (p ^ ~q) v (q
is a fallacy (contradiction). ^ ~q) or p ~q
we known that p q ( p q) ^ (q p)
13. LOGICALLY EQUIVALENT STATEMENTS :
~(p q) ~[(p q) ^ (q p)]
If truth values of statements p and q are same then they ~(p q) ~ (q p) (p ^ ~q) (q ^ ~q)
are logically equivalent and written as p = q.
Summary
14. ALGEBRA OF STATEMENTS : (i) ~ (p^ q) (~p) (~q)
(ii) ~ (p q) (~p) ^ (~q)
If p, q,r are any three statements and t is a tautology, c is
a contradiction then (iii) ~(p q)~(~p q) = p ^ (~q)
SOLVED EXAMPLES
Example 1 : Example 6 :
Find the truth values of the compound statement
Write negation of following statement:
(p v ~r) ^ (q v ~r)
“All cats scratch”
Solution :
Solution : Some cats do not scratch
OR
p q r ~r pv~r qv~r (pv~r)^(qv~r)
There exists a cat which does not scratch T T T F T T T
OR T T F T T T T
T F T F T F F
At least one cat does not scratch.
F T T F F T F
Example 2: T F F T T T T
F T F T T T T
Write negation of statement ‘2 + 2 = 7’ F F T F F F F
F F F T T T T
Solution : 2 + 2 7
Example 3 : Example 7 :
Find the truth value of the statement “2 divides 4 and Write the contrapositive of the following statement: “If
3 + 7 = 8” Mohan is poet, the he is poor”
Solution: 2 divides 4 is true and 3 + 7 = 8 is false. so given Solution : Consider the following statements :
statement is false.
p : Mohan is a poet
Example 4 : q : Mohan is poor
Write component statements of the statement “All living Clearly, the given statement is symbolic form is p q.
things have two legs and two eyes”. Therefore, its contrapositive is given by ~q ~p.
Now, ~p : Mohan is not a poet.
Solution : Component statements are : ~q : Mohan is not poor
All living things have two legs ~q ~p: Mohan is not poor, then he is not a poet.
All living things have two eyes Hence the contrapositive of the given statement is “If
Example 5 : Mohan is not poor, then he is not a poet”.
Example 8 :
Find the truth values of ~ p v q
Write the converse and the contrapositive of the
Solution : statement “If x is a prime number, then x is odd”.
p q ~p ~p v q
T T F T Solution : Given statement is : “If x is a prime number then x is
T F F F odd”.
F T T T
F F T T Let p : x is prime number and q : x is odd
Given statement is p q
The converse of p q is q p i.e., “If x is odd then x is
a prime number”
140 MATHEMATICAL REASONING
The contrapositive of p q is ~q ~p i.e., “If x is not The given statement is p and q, so its negation is
odd then x is not a prime number” ~p or ~q = Some of the students did not complete their
home work or the teacher is not present.
Example 9 :
(ii) The component statement of the given statements
Write the contradiction of “If it rains, the I stay” at are:
home. p : Square of an integer is positive.
q : Square of an integer is ngeative.
Solution : If I stay at home then. It does not rain.
The given statement is p or q. so its negation is ~p and ~q
Example 10 : = Their exists an integer whose square is neither positive
nor negative.
Let p and q stand for the statement ‘Bhopal is in M.P.’
(iii) Consider the following statements :
and ‘3 + 4 = 7’ respectively. Describe the conditional
statement ~p ~q p : My car is not in workshop
q : I can go to college
Solution : ~p ~q: If Bhopal is not in M.P. then 3 + 4 7 The given statements in symbolic form is p q
Example 11 : Now, ~ (p q) p ^ (~q)
~(p q) : My car is not in workshop and I cannot go
Find the truth value of (p ~q) (p q) to college.
Hence the negation of the given statements is “My car is
Solution :
not in workshop and i can not go to college”.
(iv) Consider the following statements:
p : ABC is an equilateral triangle.
q : It is equiangular
Clearly, the given statement in symbolic form is p q.
Now, ~ (p q) (p ^ ~q) (~p ^ q)
Example 14 : Example 16 :
By using laws of algebra of statements show that Show that the following argument is not valid : “If it
(p q) ^ ~p~p ^ q) rains, crops will be good. It did not rain. Therefore the
crops were not good”.
Solution : (p q) ^ ~p (~q) ^ (p ^ q)
(~p ^ q) (~p ^ q) Solution : p : it rains
~p ^ q S1: p q, S2 : ~p S : ~q
Example 15 : p q S1 S2 S
T T T F F
Find the negation of statement p ^ ~q
T F F F T
F T T T F
Solution : Negation of (p ^ ~q) ~(p ^ ~q) F F T T T
~ p ~~q ~p q
Not valid
142 MATHEMATICAL REASONING
(d) None of the above 11. The negation of the statement “101 is not a multiple of 3”
is
5. Which of the following is/are not a statement?
(a) 101 is a multiple of 3 (b) 101 is a multiple of 2
(I) Earth is a planet.
(c) 101 is an odd number (d) 101 is an even number
(II) Plants are living objects.
12. Which of the following is not a negation of the statement
(III) 3 is a rational number.. “A natural number is greater than zero”
(IV) x2 – 5x + 6 < 0, when x R. (a) A natural number is not greater than zero
(a) I and II (b) II and IV (b) It is false that a natural number is greater than zero.
(c) III and I (d) None of these (c) It is false that a natural number is not greater than zero.
6. The negation of the statement “A circle is an ellipse” is (d) None of the above
(a) an ellipse is a circle (b) an ellipse is not a circle 13. Which of the following is/are connectives?
(c) a circle is not an ellipse (d) a circle is an ellipse (a) Today (b) Yesteday
7. The negation of the statement “7 is greater than 8” is (c) Tommorow (d) “And”, “or”
(a) 7 is equal to 8 (b) 7 is not greater than 8 14. The component statements of the statement “The sky is
(c) 8 is less than 7 (d) None of these blue or the grass is green” are
8. Consider the statement p : “New Delhi is a city”. Which of (a) p: The sky is blue, q : The sky is not blue
the following is not negation of p? (b) p : The sky is blue, q : The grass is green
(a) New Delhi is not a city (c) Both (a) and (b)
(b) It is false that New Delhi is a city (d) None of the above
(c) It is not the case that New Delhi is not a city
(d) None of these
MATHEMATICAL REASONING 143
15. The component statement of the compound statement “If 21. The negation of the statement “If a quadrilateral is a square
you are born in some country, then you are a citizen of the then it is a rhombus”.
country” are (a) If a quadrilateral is not a square then it is a rhombus
p : You are born in some country (b) If a quadrilateral is a square then it is not a rhombus
q : You are a citizen of that country (c) a quadrilateral is a square and it is not a rhombus
Then, the correct option is (d) a quadrilateral is not a square and it is a rhombus
(a) If p is true, then q must be true 22. Negation of the statement p (q ^ r) is
(b) If p is true, then q is not true
(a) ~ p ~ (q ^r) (b) ~p (q ^r)
(c) Both p and q are false (c) (q ^ r) p (d) p ^ (~q ~r)
(d) None of the above 23. The contrapositive of the statement
16. For the compound statement “If 7 is greater than 5, then 8 is greater than 6” is
All prime numbers are either even or odd’. Which of the (a) If 8 is greater than 6, then 7 is greater than 5
following is true?
(b) If 8 is not greater than 6, then 7 is greater than 5
(a) Both component statement are false
(c) If 8 is not greater than 6, then 7 is not greater than 5
(b) Exactly one of the component statements is true
(d) If 8 is greater than 6, then 7 is not greater than 5
(c) Atleast one of the component statements is true
24. The converse of the statement
(d) Both the component statements are true.
“If x > y, then x + a > y + a” is
17. Which of the following is the conditional p q?
(a) if x < y, then x + a < y + a
(a) q is sufficient for p (b) If x + a > y + a, then x > y
(b) p is necessary for q
(c) If x < y, then x + a > y + a
(c) p only if q (d) If x > y, then x + a < y + a
(d) if q, then p
25. The contrapositive of (p ^ q) r is
18. The statement ‘x is an even number implies that x is
(a) ~r (p q) (b) r (p q)
divisible by 4’ means the same as
(c) ~r (~p ~q) (d) p (q r)
(a) x is divisible by 4 is necessary condition for x to be an
26. The converse of the statement ‘p implies q’ is
even number
(a) ~q implies ~p (b) q implies p
(b) x is an even number is a necessary condition for x to
(c) p only if q (d) ~p implies q
divisible by 4
27. Which of the following is logically equivalent
(c) x is divisible by 4 is a sufficient condition for x to be an
to ~ (p q)
even number
(a) (~p) q (b) (~p) (~q)
(d) x is divisible by 4 implies that x is not always an even
(c) p (~q) (d) p q
number
28. The converse of the statement “If sun is not shining, then
19. If p is true and q is false, then which of the following
sky is filled with clouds” is
statement is not true?
(a) If sky is filled with clouds, then the sun is not shining
(a) p q (b) p q
(c) p ^ (~q) (d) q p (b) If sun is shining, then sky is filled with clouds
20. The negation of the statement “Ramesh is cruel or he is (c) If sky is clear, then sun is shining
strict” is (d) If sun is not shining, then sky is not filled with clouds
(a) Ramesh is neither cruel nor strict
(b) Ramesh is cruel or he is not strict
(c) Ramesh is not cruel or he is strict
(d) Ramesh is not cruel and he is strict
144 MATHEMATICAL REASONING
29. The contrapositive of “If two triangle are identical, then (a) Statement I is true. Statement II is true; Statement II is
these are similar” is ...X...Here, X refers to a correct explanation for Statement I.
(a) If two triangle are not similar, then these are not identical (b) Statement I is true; Statement II is true; Statement II is
(b) If two triangle are not identical, then these are not not a correct explanantion for Statement I
similar (c) Statement I is true; Statement II is false
(c) Both (a) and (b) (d) Statement I is false; Statement II is true.
(d) None of the above 36. Let p : 25 is multiple of 5.
30. The contrapositive of the statement “If p, then q” is q : 25 is multiple of 8
(a) If q, then p (b) If p, then ~q Statement I : The compound statement “p and q” is false.
(c) If ~q, then ~p (d) If ~p, then ~q Statement II: The compound statement “p or q” is false
31. The statement “If x2 is not even, then x is not even” is Chose the correct option
converse of the statement
(a) Only statement I is true (b) Only statement II is true
(a) If x2 is odd, then x is even
(c) Both statements are true
(b) If x is not even then x2 is not even (d) Both statements are false
(c) If x is even, then x2 is even 37. The negation of the statement
(d) If x is odd, then x2 is even p : “for every positive real number x, the number x - 1 is
32. For the statement “If you drive over 80 km/h, then you will also positive” is
get a fine”, the sufficient condition is ...X...Here, X refer to (a) “there exists atleast one positive real number x for which
(a) getting a fine (x - 1) is not positive”
(b) driving over 80 km/h (b) “for every positive real number x, the number (x + 1) is
(c) Both (a) and (b) also positive”
(d) None of these (c) Both (a) and (b)
33. The statement p : For any real numbers x, y if x = y, then (d) Neither (a) nor (b)
2x + a = 2y + a when aZ
38. Negation of the statement S : “There exist a number x such
(a) is true that 0 < x < 1” is ...P... Here, P refers to
(b) is false
(a) there does not exist a number such that 0 < x < 2
(c) its contrapositive is not true
(d) None of the above (b)there does not exist a number such that 0 < x < 1
Statement II: Both the component statements of p are 39. Consider the statement
true. p : “It is necessary to have a password to log on to the
Choose the correct option server”.
(a) Statement I is true. Statement II is true; Statement II is Then, “If-then” form of this statement is
a correct explanation for Statement I. (a) If you log on to sever, then you have a password
(b) Statement I is true; Statement II is true; Statement II is (b) If you log on to server, then you do not have a password
not a correct explanantion for Statement I (c) Both (a) and (b)
(c) Statement I is true; Statement II is false (d) None of the above
(d) Statement I is false; Statement II is true. 40. If x = 5 and y = – 2 then x – 2y = 9. The contrapositive of
35. Let q: “60 is a multiple of 3 or 5”. this statement is
Statement I : q is true (a) If x - 2y = 9 then x = 5 and y = – 2
Statement II : Both the component statements of q are (b) If x - 2y 9 then x 5 and y – 2
true. (c) If x - 2y 9 then x 5 and y – 2
Choose the correct option (d) If x - 2y 9 then either x 5 or y – 2
MATHEMATICAL REASONING 145
(c) Statement–1 is false, Statement–2 is true Statement–2 : (p q) (~ q ~ p) is a tautology.
(d) Statement–1 is true, Statement–2 is true; (a) Statement–1 is true, Statement–2 is true,
Statement–2 is correct explanation for Statement–1 Statement–2 is not a correct explanation for Statement–1.
4. Let S be a non-empty subset of R. Consider the following (b) Statement–1 is true, Statement–2 is false.
statement: (2010) (c) Statement–1 is false, Statement–2 is true.
p : There is a rational number x S such that x > 0. (d) Statement–1 is true, Statement–2 is true,
Which of the following statements is the negation of the Statement–2 is a correct explanation for Statement–1.
statement P ? 8. The statement ~ (p ~ q) is (2014)
(a) There is a rational number x S such that x 0. (a) equivalent to ~ p q
(b) There is no rational number x S such that x 0. (b) a tautology
(c) Every rational number x S satisfies x 0. (c) a fallacy
(d) x S and x 0 x is not rational. (d) equivalent to p q
9. The contrapositive of the statement “I go to school if it
does not rain” is : (Online Set–1 2014)
(a) If it rains, I do not go to school.
(b) If I do not go to school, it rains.
(c) If it rains, I go to school.
(d) If I go to school, it rains.
146 MATHEMATICAL REASONING
(d) ~ Q P ~ R
MATHEMATICAL REASONING 147
20. Contrapositive of the statement ‘If two numbers are not 23. Consider the following two statements :
equal, then their squares are not equal’ is Statement P :
(Online Set–2 2017) The value of sin 1200 can be derived by taking = 2400 in
(a) If the squares of two numbers are not equal, then the the equation
numbers are equal.
2sin = 1 + sin - 1 - sin .
(b) If the squares of two numbers are equal, then the 2
numbers are not equal. Statement q :
The angles A, B, C and D of any quadrilateral ABCD satisfy
(c) If the squares of two numbers are equal, then the the equation
numbers are equal.
1 1
(d) If the squares of two numbers are not equal, then the cos A + C + cos B + D = 0
2 2
numbers are not equal.
Then the truth values of p and q are respectively :
21. The Boolean expression p q p ^ q is (2018/Online Set–2)
equivalent to : (2018) (a) F, T (b) T, F
(c) T, T (d) F, F
(a) q (b) p
24. If p ( p q) is false, then the truth values of p and
(c) p (d) q q are respectively :
22. If (p q) (p r) p q is false, then the truth (2018/Online Set–3)
values of p, q and r are, respectively: (a) F, F (b) T, F
(2018/Online Set–1) (c) F, T (d) T, T
(a) F, T, F (b) T, F, T
(c) T, T, T (d) F, F, F
148 MATHEMATICAL REASONING
ANSWER KEY
EXERCISE - 1 : BASIC OBJECTIVE QUESTIONS
1. (d) 2. (c) 3. (d) 4. (a) 5. (d) 6. (c) 7. (b) 8. (c) 9. (d) 10. (c)
11. (a) 12. (c) 13. (d) 14. (b) 15. (a) 16. (a) 17. (c) 18. (a) 19. (b) 20. (a)
21. (c) 22. (d) 23. (c) 24. (b) 25. (c) 26. (b) 27. (a) 28. (a) 29. (a) 30. (c)
31. (b) 32. (b) 33. (a) 34. (a) 35. (a) 36. (a) 37. (a) 38. (b) 39. (a) 40. (d)
1. (c) 2. (a) 3. (b) 4. (c) 5. (d) 6. (c) 7. (a) 8. (d) 9. (b) 10. (c)
11. (b) 12. (c) 13. (a) 14. (d) 15. (d) 16. (b) 17. (c) 18. (d) 19. (c) 20. (c)
21. (b) 22. (b) 23. (a) 24. (d)
Dream on !!